sábado, 25 de abril de 2015

Acerto

Prof.sofri um acidente há 5 dias, no qual minha tia veio a morrer.Foi uma experiência traumatizante, mas ñ me aproximei da religião por conta disso.Contudo, utilizo da fé q meus familiares possuem para consolá-los.Estou agindo errado? Existem palavras de consolo nesses momentos q dispensem a fé? obg‎

Sim. Não é preciso fazer referencia nenhuma a uma pretensa vida da alma após a morte para consolar os parentes e amigos de quem morreu. Pode-se discorrer sobre as virtudes do morto (ou morta), de seu exemplo de vida, de como ele fez o mundo ficar melhor por ter vivido, de como suas lições serão proveitosas para todos que ficaram, de como será doce sua recordação. Não precisa e nem se deve, nesse momento, contestar quem aborde questões religiosas. Simplesmente não se dá corda a essa linha de pensamento e se desvia para uma linha humanista não religiosa.

Acredita em ovnis e ets?

Ovnis não são passíveis de se acreditar ou não. Eles se apresentam à observação e são chamados de ovnis se não se identifica de que se tratam. Ao serem identificados, deixam de ser ovnis. O que requer alguma crença é a suposição de que os ovnis sejam objetos de origem extraterrestre, especificamente, naves produzidas por seres inteligentes extraterrestres. Quanto a isso não há nenhuma comprovação. Portanto, considerar que o sejam é, meramente, uma crença. E uma crença sem fundamento. O mesmo pode-se dizer sobre seres vivos extra-terrestres, especialmente inteligentes. Não há nenhuma comprovação de que existam, de que tenham nos contactado ou, pelo menos, tentado se comunicar conosco. Isso não descarta a possibilidade de existirem. Penso que possam existir. Mas não acho que se comuniquem conosco. Todavia também pode ser que não existam e que sejamos únicos no Universo. É só o que podemos garantir.

Qual foi o primeiro individuo vivo que teve o sexo diferente do seu antecessor?‎

Leia isto:
http://pt.wikipedia.org/wiki/Evolu%C3%A7%C3%A3o_do_sexo
http://en.wikipedia.org/wiki/Evolution_of_sexual_reproduction
http://es.wikipedia.org/wiki/Evoluci%C3%B3n_de_la_reproducci%C3%B3n_sexual
http://www.allaboutscience.org/evolution-of-sex.htm

Professor, sou apaixonada por física e química e pretendo fazer faculdade de física, mas tem um porém, não sou muito bom com calculos, isso seria um problema muito grande ou eu posso aprimorar isso com o tempo?‎

A Física, realmente, requer muita matemática. Penso, até, que mais do que o próprio curso de Matemática. Sem Matemática a Física não se desenvolve. Só que não é apenas Matemática. É preciso, também, uma compreensão fenomenológica do que está acontecendo em cada caso. Isso faz a Física, em geral, ser considerada mais difícil do que a Matemática. Não acho que seja. Quem tenha pouca facilidade para Matemática precisa adquiri-la. Ao longo do Ensino Médio isso pode ser feito, se a pessoa se dispuser. Mas só vai conseguir se tiver gosto. Estudar Matemática por obrigação não produz bons resultados. A Matemática é a linguagem por meio da qual os fenômenos físicos são modelados de forma a poderem ser controlados.

Qual é a propriedade que faz a luz ser tão rápida?‎

A permissividade elétrica e a permeabiidade magnética do vácuo. Essas propriedades é que definem, na equação da onda eletromagnética, a rapidez com que os campos elétricos e magnéticos oscilantes sejam capazes de se auto-propelir, criando a onda. Isso foi deduzido por Maxwell, que previu que tais ondas deveriam existir. E foi experimentalmente confirmado por Hertz. O interessante é que a velocidade deduzida por Maxwell foi exatamente o valor conhecido da velocidade da luz. Como já se sabia que a luz era uma onda, concluiu-se que era uma onda de campos elétricos e magnéticos. Só que não se conseguia desenvolver um dispositivo elétrico capaz de produzir luz. O que Hertz produziu foram microondas. Só com o advento da Física Quântica é que se compreendeu o mecanismo elétrico de emissão da luz, a partir da desexcitação de átomos excitados.

Professor, esse papo de egoísmo e ciúme me faz lembrar do meu pai. Até os 17 anos ele não me deixava nem ir na casa de amigas. Sem contar que ele me achava incapaz de fazer coisas que meninas de 12 anos já fazem, como pegar ônibus. Tudo alegando ciúmes e preocupação.Você acha que isso é amor?‎

De modo nenhum. Isso é possessividade. Algo horrível. Quem ama, confia. Quem ama, liberta. Quem ama, delega. Quem ama quer a felicidade do amado. Quem ama não controla, não vigia, não fiscaliza. E se o amado usar essa liberdade para amar a outrem? Qual o problema? Porque não querer que quem amamos possa amar a quem queira amar. Amor não se pode exigir, porque, inclusive, não adianta nada. Exigir amor não garante amor, de modo nenhum. O que se pode é desejar ser amado e fazer por onde merecer ser amado. Isso, certamente, exclui qualquer exigência e qualquer policiamento. Quem tem ciúme, além de uma pessoa egoísta é cruel é uma pessoa burra.

Por que dizem que as estrelas que vemos são uma fotografia do passado?‎

Porque a luz que traz a sua imagem que vemos saiu delas há tempos atrás, viajando a trezentos mil quilômetros por segundo. Vemos a Lua a pouco mais de um segundo atrás. Vemos o Sol há oito minutos atrás. A mais próxima estrela é vista há quatro anos atrás. O centro da Via Láctea é visto há mais de vinte mil anos atrás. A galáxia de Andrômeda é vista a dois milhões e quinhentos mil anos atrás. Os quasares mais distantes são vistos a bilhões de anos atrás.

Tratar as pessoas como você quer tratado/a ou tratá-las como elas te tratam?‎

Como eu quero ser tratado. Sem a menor dúvida. Eu quero ser tratado com respeito, consideração, educação, cortesia, apreço, carinho, fidalguia. Então é assim que eu trato os outros, mesmo que não me tratem assim.

Infelizmente a realidade é que os alunos não conseguem interagir por conta própria dentro do assunto proposto. O foco se perde facilmente. É preciso que haja um professor para colocar ordem na sala.‎

Claro que sim. Mas nem sempre muita ordem é bom. O importante é a motivação. Fazer com que os alunos queiram trabalhar e o façam com gosto. O professor tem que ser muito entusiasmado com sua matéria. E muito entusiasmado com a atividade pedagógica. Se não for, desista de ser professor.

Quem entra(r) num buraco negro morre ou vai pra outra dimensão?‎

Morre.

Por q a física eh mais difícil que a matemática?‎

Porque requer a identificação do fenômeno que está ocorrendo, de suas circunstâncias e, a partir daí, a identificação do modelo teórico que se aplica e dos valores das grandezas pertinentes. Não é só pegar a fórmula e fazer as contas. Tem que saber que fórmula usar, em função da situação. Isso requer uma compreensão muito boa do funcionamento da natureza. Além de requerer, também, o domínio da matematica a ser aplicada no caso.

Professor, dou aulas para turmas que querem se preparar para o ENEM paralelamente às aulas escolares, e, nas minhas aulas, há muita movimentação de aluno indo para o quadro ou formando grupos para resolverem e debaterem questões e exercícios. Isso é bom ou não?‎

Claro que é bom. Quanto mais ativa e participativa a aula, melhor. Aula não é palestra. Não é um momento em que alguém fica falando, falando e a turma ouvindo, ouvindo. Aula tem que ser uma atividade dos alunos. Eles trabalhando, eles descobrindo, eles tentando, eles interagindo entre si. Pode haver desvio para interações paralelas fora do contexto da aula. Não tem problema se for num nível tolerado. Mas é preciso que o professor mantenha a turma bem ligada nas atividades, que têm que ser muito motivadoras.

Professor Ernesto, se o universo não observável for infinito, o universo observável surgiu no interior daquele. Então o que impediria o surgimento de infinitos universos "observável"? Sei que em ciência há de se ter provas ou fortes evidências. Mas me parece mais do que óbvio tais ocorrências.‎

Note que os Universos Observáveis não surgem. Eles são parte do Universo. São a parte que se pode observar de dado ponto, para o qual a luz teve tempo de chegar desde que o Universo existe. Existem infinitos Universos Observáveis mesmo. De cada lugar do Universo em que se está, o Universo Observável para esse lugar é uma região esférica de raio igual à distância ao ponto de observação em que se situa algo que enviou sua luz para o ponto de observação quando o Universo surgiu e ela chegou ao ponto de observação no momento presente, tendo centro no ponto de observação. Atualmente esse raio, considerando o tempo que o Universo surgiu, que é de 13,8 bilhões de anos, e o fato de que, enquanto isso, o Universo foi se expandindo, é da ordem de 46 bilhões de anos-luz. Veja o cálculo aqui:
http://dournac.org/info/size_universe
http://physics.stackexchange.com/questions/57402/size-of-the-observable-universe

Costumam chamar pessoas que se importam muito com a aparência e menos com o conhecimento de fúteis. Você concorda?‎

Sim, concordo. A questão não é, especialmente, o conhecimento, mas o que seja relevante, como o caráter, as virtudes, a educação. Não que se deva desleixar da aparência, mas que ela não pode ser considerada como o valor mais elevado. O mesmo se aplica à reputação, à riqueza, ao poder, a fama.

Estava lendo suas respostas sobre ciúme e me questionando: Dá para amar alguém sem ser egoísta, querer ela somente para si? Seres humanos são egoístas...‎

Mas é claro que dá. Perfeitamente. Os seres humanos podem ser egoístas, mas não são necessariamente egoístas. Há quem seja naturalmente altruísta e quem for naturalmente egoísta pode deixar de ser, se educando para isso. Não vejo nenhum problema em compartilhar meu amor com outra pessoa. Isso não significa, em absoluto, que meu amor seja nem um tiquinho menor. Da mesma forma que eu sou capaz de amar a mais de uma outra pessoa com amor sincero, intenso, completo e profundo por cada uma delas. Isso é um fato perfeitamente constatável.

Ernesto, existe algum cálculo matemático ou outra forma de comprovar que o campo existente antes do Big bang, que, dizem, encontrava-se concentrado em um ponto do tamanho de um elétron, tenha gerado todo o conteúdo do universo observável?

O que se concentrava no volume de um elétron não era o Universo e sim o Universo Observável, que é a parte dele que se consegue detectar, pois o que está além a luz ainda não teve tempo de chegar até nós desde que o Universo existe. E o Universo, ao que tudo indica, é infinito. Sendo assim, ele sempre foi infinito, desde que surgiu. O que aconteceu é que os lugares se afastaram uns dos outros com a expansão cósmica. Cálculos matemáticos não comprovam nada. Eles preveem. O que comprova é a observação e a experimentação. Todavia há cálculos que permitem obter o fator de expansão do Universo desde o início até hoje, com base na equação de Friedmann. Só que ele não leva em conta a "inflação cósmica" que ocorreu logo no início e provocou um inchamento colossal. Levá-la em consideração é bem complicado. Mesmo sem ela, o cálculo é complicado e não dá para ser exposto aqui. Vou procurar se tem algum artigo na internet que o desenvolva e postarei depois.

você não acha injusto uns viverem 100 anos e outro não viverem nem 1?por isso às vezes fico mocado pensando que possa existir mais de uma vida,ou até infinitas..‎

O fato de ser injusto não acarreta que alguma justiça tenha que ser feita com a concessão de outras vidas. Justiça é um conceito social que surge em populações de seres dotados de inteligência e consciência, como os humanos. Na natureza não há justiça nem injustiça. Há fatos. A consideração sobre a injustiça da diferença de oportunidades, inclusive biológicas, é de natureza cultural, engendrada por nossa espécie (ou outra inteligente que houver). Não há nenhuma necessidade inerente ao mundo natural de que se faça justiça a respeito do que quer que seja. Esse desejo é humano.

Curte scat Ernesto?

Sim. Faço muito scat. Invento a melodia, a letra e o idioma. Não significa nada. Vou cantando. Adoro isso. Mas não é só Jazz. Gosto de scatear qualquer tipo de música, até mesmo ópera e música sacra. Gosto de sílabas do tipo da língua russa. Ou os idiomas escandinavos. Ou sons italianos. Ou franceses.

você acha a vida injusta?você acha justo uns terem poucos,outros terem muito?‎

Claro que essas diferenças são injustas. Uma sociedade justa é aquela que ninguém tem nada. Tudo é de todos. Tudo é compartilhado. Todos trabalham uns pelos outros. Ninguém é mais pobre nem mais rico que ninguém. Isso é o comunismo. Mas um comunismo comunitário. Coletivista. Nada do governo ser o patrão geral. Aliás, nada de governo. Governo nenhum mesmo.

se "Deus" é tão bom,então porque muitas crianças não tem nem direito de viver?porque muitas morrem antes de completar um ano?serio eu n entendo...‎

Já está mais do que provado que, se Deus existir, absolutamente não é bom. O mais plausível é que não exista mesmo.

Prof, é possível esquecer propositalmente de uma informação? Sem querer li o final de um seriado que assisto. É possível esquecê-lo?‎

Voluntariamente não. Quanto mais você se esforçar para esquecer, mais se lembrará. O esquecimento só acontece espontaneamente. Mesmo assim, se algo relacionado com o assunto for apresentado, a memória será evocada.

Qual sua opinião sobre o feminismo? Acha válido atualmente?

Completamente válido. É a única postura eticamente aceitável sobre o assunto. Mas não se pode confundir feminismo com femismo, que é o que muitos que se dizem feministas defendem. Femismo é a defesa de privilégios para mulheres. Isso é abominável. Feminismo é a defesa de direitos e deveres iguais para homens e mulheres.

Professor o que acha de físicos que dizem que espírito existem através da física quântica? Há vários vídeos no youtube sobre o assunto, já vi você falando sobre espíritos e fiquei intrigado com esses vídeos. Eles usam até a ressurreição de Jesus como um fato na física quântica.‎

Isso é a mais completa baboseira. Não tem nada na Física Quântica que confirme a existência de espíritos. A ressurreição de Jesus é uma lorota. Uma invenção. Um golpe de merchandising. Essas pessoas não são legítimos cientistas. Elas entendem de Física mas usam a Física para justificar suas crenças, forçando os fatos. Um cientista é sempre um livre pensador que jamais parte de convicções pessoais para construir suas explicações, mesmo que as tenha.

Professor, vc acha mais coerente dizer que nascemos ateus, ou agnósticos?‎

Ateus. Porque o agnosticismo é uma postura muito sofisticada que requer uma reflexão sobre a possibilidade ou não de se saber da existência ou não de Deus. O ateísmo é, simplesmente, a concepção da inexistência de Deus. Isso é muito mais simples do que a concepção de sua existência, que requer um aprendizado do significado de seu conceito. Uma criança pequena não tem condições de assimilar o que seja Deus e, portanto, de considerar que existe. Uma vez que ela nem sabe do que se trata, ela não acha que existe, sendo, portanto, ateia.

O fato de os planetas e estrelas serem "arredondados" tem alguma relação com a pressão exercida pelo universo?

Não. O formato esférico é decorrente do fato que a força que aglutina o material que forma o planeta, a gravidade, é uma força central atrativa. Então a forma capaz de minimizar a energia potencial de um sistema sujeito à gravidade recíproca de suas partes é a forma esférica. Como os planetas, quase em sua totalidade, também exibem rotação, o efeito centrífugo promove um achatamento nos pólos e uma dilatação no equador. O fato de haver rotação advém de que o material que se agrupa e forma o planeta (ou a estrela) possui componentes de velocidade tangencial à direção radial de aglutinação, que quase sempre não são compensadas, fornecendo um momento angular não nulo ao sistema.

sexta-feira, 24 de abril de 2015

Professor, no fundo do mar, por exemplo à 2000m de profundidade, a pressão que é exercida sobre um corpo vem de todas direções? Como se estivesse apertando uma esfera?‎

Não só no fundo do mar como dentro de qualquer fluido em um campo gravitacional, a qualquer altura ou profundidade.

Qual a sua resposta aos que dizem que você rotula muito a si mesmo (anarquismo, vegetarianismo, ateísmo, comunismo, epicurismo, etc.)?

Que esses rótulos são aproximações e não engessamentos. Em relação ao vegetarianismo, acabei me convertendo ao veganismo. Mas admito comer algum produto de origem animal, dependendo das circunstâncias. Do mesmo modo em relação a tudo o mais. Sou comunista, mas posso aceitar algo que seja considerado capitalista. Da mesma forma quanto ao anarquismo. Se me ficar provado que exista algum tipo de entidade sobrenatural, poderei deixar de ser ateu. O importante, em tudo, é ser aberto a mudar as concepções e convicções. Isto é, ser um livre pensador. Mas sempre ser aquilo de que se está convencido, até que se mude. E se convencer sempre do que for completamente razoável e demonstrado de forma cabal.

e porque você não entra para política? Lá você pode ajudar para com que se aproxime a anarquia através de evolução política‎

Porque não tenho vocação. Minha vocação é pedagógica e científica. Quero é difundir idéias, convencer as pessoas. Além do mais, acho que iria enfartar pela terceira vez na política. Porque não admito nem um tiquinho de nada de concessão a qualquer safadeza. Os próprios eleitores não iriam querer quem não lhes concedesse nenhum tipo de regalia e só prometesse "sangue, suor e lágrimas". Mas é isso que é preciso para consertar o mundo.

Bandidos tinham que ser fuzilados em vez de ficar engordando na cadeia com o dinheiro do meu imposto. Bando de marginais sem cultura, sujos, pobres e, para piorar, sem nenhuma perspectiva de vida; não fariam nenhuma falta para a humanidade. Nem tinham que ser considerados humanos. São bichos!‎

DISCORDO! Bandidos são pessoas humanas. Não se pode matar pessoas, exceto em legítima defesa, no momento da ameaça. Criminosos têm que ser presos e cumprirem pena rigorosa, com trabalho para gerar renda que pague parte de sua hospedagem carcerária. Sem redução de pena por razão nenhuma. A finalidade do encarceramento é tripla: segregar a pessoa da sociedade, para protege-la, punir o crime para que não compense e regenerar o criminoso. Além do mais, pena de morte não permite corrigir erros judiciais.

Conhece a taxonomia de Bloom ? É uma excelente metodologia "mensuradora" do conhecimento.‎

Sim. E a uso para planejar meus cursos e suas avaliações desde que comecei a lecionar em 1968. Inclusive na forma de abordar os assuntos em classe. Com algumas modificações introduzidas por mim mesmo. Nas habilidades do domínio cognitivo eu incluo o entendimento que distingo da compreensão. O entendimento é saber as razões e os modos, enquanto a compreensão é saber como o assunto se coloca dentro do conjunto dos demais conhecimentos. Na formulação de uma avaliação eu construo uma matriz (em verdade uma matriz paralelepipédica) com eixos para os conteúdos, os objetivos e os níveis de complexidade. Então elaboro as questões para atender os requisitos elencados. Nunca coloco questões tiradas de livros ou outras avaliações. Sempre invento todas.

Se um dia eu me tornar alguém importante na ciência, posso dizer que uma das minhas fontes de inspiração foi seu conhecimento compartilhado?‎

Se, de fato, for isso, claro que pode dizer. Fico feliz em ler isso.

Você gostaria de morar em Cuba se pudesse?‎

Não. Não só porque não aprecio nem um pouco o regime cubano mas, porque, mesmo que fosse bom, não é o tipo de cultura que eu goste. Se for para sair do Brasil, preferiria ir para a escandinávia, Áustria, Suíça, Holanda, Bélgica ou similares. Mas prefiro o Brasil, não porque concorde com tudo como está aqui, mas porque quero ajudar a melhorar.

você concorda que deus não vai morrer até que os representantes da ciência digam o que veio antes do nada? É que, penso eu, que pra pessoas completamente leigas no assunto, a explicação da biologia é tão absurda quanto a bíblica‎

Discordo. A explicação científica não é nada absurda. A bíblica e a de outras mitologias é que são absurdas. São explicações gratuitas e sem fundamento. A ciência, quando não sabe, diz que não sabe e que ainda não se tem explicação. E a busca de modo sistemático e persistente. Sem inventar nada. Antes reconhecer a ignorância do que propor explicações fantasiosas. Quem for leigo no assunto que se debruce sobre ele e deixe de ser leigo. Todo mundo pode fazer isso. Por outro lado, quem acha que as mitologias religiosas explicam o surgimento do Universo e da vida, se enganam. Não há explicação nenhuma sobre o modo como se alega que Deus tenha procedido para fazer à criação. É incoerente os religiosos exigirem que a ciência dê explicações se eles não dão. E nem as buscam, enquanto a ciência está sempre investigando para explicar. Acham que querer saber como Deus fez o que fez é falta de respeito. Tem que aceitar sem questionar. Isso é que é absurdo.

o que tu mais odeia nas pessoas?‎

O egoísmo e o que dele decorre: ganância, preguiça, impiedade, crueldade, esnobismo, soberba e coisas do tipo.

Ernesto percebo que gosta de ajudar. Mas fora ao ask, qual tipo de ajuda que gosta de fazer? Por quê? Qual o objetivo de ajudar?

Ajudo as pessoas em tudo que puder. Não só passando conhecimentos, como também financeiramente, e em trabalhos braçais. A razão é que, assim o fazendo, estou contribuindo para que o mundo se torne um lugar melhor, mais aprazível, mais justo, mais bonito. E isso me enche de satisfação. Mesmo que não seja reconhecido e não tenha retribuição, não me importo. Fico feliz em ser útil. Gosto disso. Isso é uma índole natural minha. Mas também é um exemplo que assimilei de meus pais e, em geral, de toda a minha família, tanto materna quanto paterna. São todos altruístas, desprendidos, generosos, diligentes, prestativos. Sempre achei que isso era o normal de todo mundo. Só muito tarde é que fui ver que existe gente mesquinha, egoísta, preguiçosa, gananciosa. Achei muito estranho e, até hoje, não entendo como alguém possa ser assim.

É possível uma pessoa ficar "maluca" com excesso de informação ou conhecimento no cérebro? Adquirir informação demais é saudável?‎

Não é possível. O cérebro pode conter mais informação do que se é capaz de absorver no tempo de vida, mesmo que se viva 150 anos. Não tem perigo nenhum adquirir informação demais. Nunca é demais. É saudável ter muito conhecimento sim. É muito bom. Pode mergulhar no oceano do conhecimento com toda segurança. Não vai se afogar.

Na sua opinião, por que os religiosos atacam muito os roqueiros?Sou roqueiro, não sigo as ideias satânicas, mas já fui discriminado várias vezes por religiosos fanáticos por eu ser roqueiro, o que você acha dos religiosos que atacam tudo que defende a liberdade?‎

Em geral, pessoas religiosas são infensas à liberdade, pois liberdade inclui, também, poder contestar as religiões, como tudo o mais. Rock, normalmente, é contestador. Contestador dos costumes tradicionais, das concepções retrógradas, das práticas rotineiras, da intolerância, dos preconceitos. Religiosos são conservadores. Querem tudo fique como está, isto é, de modo a confirmar suas concepções. Esquecem que Buda, Jesus, Maomé, Lutero foram grandes contestadores revolucionários. Os fundadores das igrejas neo-pentecostais de hoje se rebelam contra as outras formas de cristianismo, mas não admitem que se rebele contra a forma deles. O que é preciso é se rebelar contra toda e qualquer religião e se tornar um livre pensador. Isso não significa ser uma pessoa sem ética e sem princípios. Significa ser livre. Curta o Rock e esqueça a religião. E não se importe com o que digam de você. Mas seja íntegro, justo e coerente. Além de, é claro, uma pessoa bondosa e de bem.

Qual a diferença entre conhecimento e informação?‎

Conhecimento é informação entendida, compreendida, contextualizada, classificada, justificada, comprovada,

de onde vieram as matérias que estavam comprimidas antes da explosão que originou o universo?‎

O que estava comprimido antes do Big Bang não era matéria, era campo. Com a expansão é que o campo se quantizou em matéria e radiação, além de continuar como campo também. Esse campo não proveio de lugar nenhum. Ele surgiu sem ter do que e de onde provir. Pois, não havendo Universo, não haviam lugares em que algo pudesse estar e nem conteúdo nenhum de que algo pudesse surgir. Surgiu de nada, isto é, sem ter de que provir.

O fato de podemos ir pro futuro significa que as coisas já estão predestinadas?‎

De modo nenhum. Tudo vai ocorrendo à medida que os eventos vão se dando. Não existe predestinação. O fato de se poder ir ao futuro em ritmos distintos significa que os eventos que as pessoas que vão a esses ritmos diferentes vivenciam são outros. Porque, ao se ir ao futuro, também se move pelo espaço. Quem sente o tempo ir mais rápido não está nos mesmos lugares de quem o sente ir mais lentamente. Só quando voltarem a estar num mesmo lugar é que verificarão os ritmos distintos de tempo que sofreram. Leia isto:
https://pt.scribd.com/doc/252163421/Paradoxo-dos-Gemeos

Em uma anarquia teria circulação de moeda?‎

Depende do modelo de anarquia que se considerar. Pode haver ou não. Mesmo que a anarquia seja comunista. Para mim, não deve haver. Nem moeda nem propriedade.

Falta originalidade e também incentivos por parte dos professores de Filosofia do Brasil. Se você tentar criar algo novo (uma tese), numa monografia, por exemplo, eles dizem: "é complicado tratar isso numa monografia", "faça-o em seu doutorado!", "você não vai conseguir".Como lutar contra o sistema?‎

Luta-se contra o sistema insistindo e sendo brilhante. Tão brilhante que não há como ser recusado. Sendo muito melhor do que os professores. Assim é que se derrota o sistema. Mas não pode ser arrogante, presunçoso, esnobe, sarcástico. Nada disso. É preciso ser assertivo, isto é, positivamente modesto. Sem se humilhar e sem se jactar. Vencendo por argumentos, pela dialética, pela lógica, pela retórica, pela inteligência, pela fundamentação. Com ousadia, contudo. Sem medo.

Professor, por que conseguiríamos ir para o futuro se criássemos uma nave com velocidade superior a da luz?‎

Não precisa isso. Nós estamos sempre indo para o futuro. A questão é ir para o futuro mais rapidamente do que o resto. Isso acontece com qualquer coisa que se mova, a qualquer velocidade. À velocidade da luz, o tempo não passaria. Então se iria imediatamente para qualquer tempo futuro. Mas isso não tem como acontecer com nada material. Não há como se ultrapassar a velocidade da luz. Nem atingir. Só mesmo a própria luz (e todas as ondas eletromagnéticas).

"A maior parte da massa dos hadrons é devida à energia dos glúons que existem dentro deles." Isso é devido a aquela famosa fórmula E = m c²?‎

Sim, os quarks dos prótons, entram apenas com 11 MeV/c² para a massa, enquanto a energia dos glúons entram com 927 MeV/c². Veja isto:
http://en.wikipedia.org/wiki/Quark#Mass

não perca tempo procurando saber como o mundo surgiu,viva a vida.‎

Pois eu não perco tempo. Eu "dispendo" tempo. E isso é a minha forma de viver a vida, pois, para mim, o que me dá mais prazer, depois do amor, é o conhecimento. Buscando conhecimento é que estou vivendo plenamente a minha vida. E o que eu gosto de descobrir é, justamente, tudo o que não seja simples. Isso é que me dá a maior satisfação. Mesmo que eu não ache a resposta, só o trabalho de procurá-la me enche de prazer. Adoro complicações.

O mundo está se tornando mais religioso. O que você pensa sobre isso? Fonte: http://www.livescience.com/50370-worlds-religious-population-will-grow.html?1428124826030=1‎

Que pena! Espero que, depois de 2050, essa tendência se reverta e as religiões todas percam adeptos. Isso será o resultado do aumento da prosperidade e da educação. Especialmente no mundo islâmico é preciso que o crescimento seja revertido, pois esta é uma religião bem nefasta, em razão da maior incidência de fundamentalistas. Gostaria de saber como se projeta esse crescimento regionalmente, pois penso que o aumento global seja devido, principalmente, à África, Ásia e América do Sul e Central, com diminuição na Europa, América do Norte, China, Japão e Oceania. Para reverter esse crescimento será preciso que a prosperidade desses lugares aumente muito e que o nível de educação formal também. Isso ainda leva alguns séculos. Além, é claro, da diminuição da taxa de natalidade.

http://ask.fm/wolfedler/answer/125797885981 e isso é possível, não vir de lugar algum? como é possível?‎

Se não for possível, então o Universo tem que ter sempre existido. Isso é possível? Tanto a opção de ter sempre existido quanto a ter ter surgido sem ter de que provir contrariam nosso senso comum. Mas uma delas tem que ser verdadeira. Todas as informações sobre o Universo conduzem à conclusão de que ele nem sempre existiu. A explicação sobre como se deu o surgimento de tudo sem provir de nada não é conhecida. Do mesmo modo que quem considera que o Universo tenha sido criado por Deus não sabe explicar como ele teria feito isso sem ter de onde tirar o conteúdo do Universo. Se se pode aceitar que esse Universo tenha sido criado por algum agente sem ter de que ser proveniente, porque não aceitar que tenha surgido sem ter sido criado, também não tendo de que provir? Por outro lado, se existia um Deus, então havia algo e como o Universo é o conjunto de tudo o que existe, havia Universo, que era o Deus.

O que você pensa sobre as coisas que o Apostolo Valdemiro Santiago diz?‎

Que apóstolo? Valdemiro Santiago é um charlatão. Penso que ele só diz besteira e só quer saber de enganar o povo para se locupletar. Tinha é que estar na cadeia.

A grade curricular do curso de filosofia me parece incoerente. Essa ênfase na historia da filosofia me parece despropositada. Dane-se a cronologia. Acho que a preocupação maior deveria ser levantar as grandes questões da filosofia com o auxílio do conhecimento científico moderno. O que pensa?‎

Esta é. realmente, uma grande falha. Os cursos de filosofia não formam filósofos e sim entendidos em filosofia. Comentadores de filósofos, mas não, eles mesmos, filósofos. Em um curso de música ou de artes plásticas se formam artistas. Claro que é preciso ter um talento inato. Mas o curso dá as ferramentas necessárias para que o talento desabroche. Nos cursos de letras também há essa falha. Não se formam escritores e nem poetas. Só entendidos do idioma e sua literatura. Podem formar linguistas. Mas os de filosofia nem isso. Uma lástima. Claro que se precisa conhecer a História da Filosofia. Mas isso, que era para ser uma condição básica, passa a ser a própria finalidade do curso. Parece que os professores têm medo de ousar, de pensar por conta própria. Ora, os grandes filósofos só foram grandes porque ousaram contrariar tudo o que estava estabelecido. Esse medo é o que se chama "complexo de vira-lata".

O senhor acha que vale a pena estudar numa escola militar? Ou da marinha, como a EFOMM?‎

Não conheço a EFOMM, mas acho as escolas preparatórias militares muito boas. Já as de formação de oficiais (nível superior) não acho interessantes, pois são profissionalizantes para a vida militar, que não aprecio nem recomendo. Só se você, realmente, quiser ser militar. O trabalho na Marinha Mercante acho que deva ser bom.

Pra onde vão os elétrons removidos de uma placa metálica devido ao efeito fotoelétrico?‎

São ejetados para o espaço circundante à placa.

Segundo tua opinião da onde veio o Universo?‎

Não vei de lugar nenhum. Pois, se houvera algo de que o Universo fora proveniente, então isso já seria Universo.

Por que pessoas ruins vivem muito e pessoas boas vivem pouco?‎

Isso não é verdade. Não há nenhuma correlação entre o tempo de vida e a maldade ou bondade de uma pessoa.

Conhece Ademar José Gevaerd? O que acha dele?‎

Sei que é um ufólogo, mas nada mais. Não tenho particular interesse por ufologia.

Na natureza há casos de coisas que vem a existir sem ter do que provir ?‎

Depois que já existe algo, o que existe se comporta de acordo com certas ditas "leis", descritivas desse comportamento. Dentre elas estão as "leis de conservação", que descrevem como os processos naturais se dão com relação a certos atributos dos sistemas, cujas grandezas que quantitativamente os descrevem, são mantidas invariáveis. Dentre elas o conteúdo de massa-energia, a carga elétrica, os momentos linear e angular e outras. Assim, o surgimento "de nada" não se daria em atendimento a tais leis, não sendo pois, observado. Se o for, contudo, as leis têm que ser revistas. O surgimento do Universo, por outro lado, não precisou ter acontecido com o atendimento de nenhuma lei, pois, nada existindo, também não há lei a ser aplicada. Portanto o surgimento de tudo sem ter de que provir só pode ter ocorrido uma vez. O que acontece, atualmente, é o surgimento de pares de partícula e anti-partícula. Mas eles são provenientes do campo do vácuo, que é algo e não nada.

Já se tem alguma ideia de como algo pode vir de nada ? alguma hipótese nem que seja apenas uma tentativa de explicação.‎

"Vir" de nada não pode acontecer. O que pode acontecer é algo surgir sem "vir" de coisa alguma. Quando se diz que algo "veio", isso significa que, havia algo, em algum lugar, de que o que está se considerando proveio, em algum processo de transformação. Surgimento sem ter do que provir é outro fato. É um surgimento que não se configura em transformação de algo em outro algo. Nesse caso não há coisa alguma a partir da qual o que surgiu tenha surgido. O surgimento aconteceu sem proveniência de nada. Como é que tal feito pôde acontecer não se sabe. Isso, justamente, é objeto de grande especulação e intensas pesquisas. Ainda não se tem hipóteses a respeito. Isso é que é explicar o surgimento do Universo. Não é o Big Bang. O Big Bang é o fenômeno de expansão do Universo e não do surgimento. O fato de ainda não se saber como surgiu o Universo, contudo, não autoriza a considerar que seu surgimento tenha sigo provocado pela interveniência de alguma pretensa entidade extrínseca a ele, que seria Deus. Por outro lado, mesmo que assim o fora, esse Deus fez surgir o Universo sem ter do que provir. A interveniência de Deus só altera o fato do surgimento ter tido uma causa ou não. Mas não altera o fato dele ter se dado sem ter do que provir.

Você disse que o nada não existe mas o universo veio de nada não é verdade ?‎

Vir "de nada" não é o mesmo que vir "do nada". Se se disser que o Universo veio "do nada", está se dizendo que o Universo veio de algo que seria "o nada". Se se disser que o Universo veio "de nada", está se dizendo que o Universo não proveio de coisa alguma. Este á o caso.

Opine sobre a redução da maioridade penal. Abraços, e queria parabenizá-lo pelas ótimas respostas.‎

Digite no Google:
inurl:wolfedler maioridade penal

Alma não existe? Por exemplo se vc for clonado por cientistas, o clone vai ser exatamente idêntico a vc, mas não ira ser vc porque não é sua "alma" ali dentro , vc não vai controlar aquele corpo. Ou isso é pura bulhufas ?‎

É pura bulhufas. Um clone meu não será eu não é porque não tem a minha alma. É porque não viveu a minha vida. Se no clone for implantada todas as minhas memórias então ele será outro eu. Mas a partir do momento em que eu e ele continuarmos a viver nossas vidas distintas, passaremos a ser seres diferentes, com a memória de um passado compartilhada.

O "Nada" existe? Ou todo lugar está sendo ocupado por algo?‎

Nada não existe. Não é algo e sim a ausência de qualquer coisa. Inclusive de espaço, mesmo vazio. Também não existe espaço vazio no Universo. Todo espaço existente é ocupado, pelo menos por campo e radiação. Se não houver matéria no espaço ele é um vácuo. Mas nunca é vazio. Certamente que fora do Universo não existe nada. Nem conteúdo nem espaço. Aliás, não existe "fora do Universo". Tudo o que existe está no Universo.

Mas não existem várias fotos que mostram as linhas de força de um campo nos livros?Como são meras representações?‎

Não são fotos de linhas de força. São fotos de indicadores de campo. No caso do campo elétrico eles podem ser sementes de grama. No caso de campo magnético podem ser limalha de ferro. Os indicadores são pequenos objetos finos e compridos sobre os quais o campo aplica um torque que os posiciona alinhados com ele mesmo. Assim, a coleção dos indicadores passa a apontar na direção e sentido que o campo aponta, que é justamente como são definidas as imaginárias linhas de força que, em verdade, são linhas de campo e não de força.
http://www.cidepe.com.br/assets/img/content/products/conjunto_guimaraes_fisica_geral_eq060_3.jpg
http://www.wikienergia.pt/~edp/images/8/86/Linhasmagneticasforca.jpg

Alma existe ?

Não.

Qual é a diferença entre "faculdade" e "universidade"? Quais são as origens dessas palavras? Essas palavras, hoje, tem o mesmo significado que tinham no passado?

Faculdade é uma escola que tem cursos de nível superior. Universidade é um conjunto de faculdades sob uma mesma administração. Leia isto:
http://origemdapalavra.com.br/site/palavras/faculdade/

Do que são feito os pensamentos?‎

Pensamentos não são substanciais. Não são feitos de nada. São ocorrências, acontecimentos, eventos. Eles se dão em razão das interações interneuronais. Essas interações acontecem por transmissão de inversão de polarização da membrana celular nos dendritos e axônios e por transportes de neurotransmissores nas fendas sinápticas entre os dendritos e os axônios. A ocorrência de um conjunto de mensagens entre neurônios vem a ser um pensamento.

Você concorda com o processo seletivo para a admissão de alunos nas universidades brasileiras (provas, basicamente)? Como seria o processo ideal?‎

O ideal seria uma avaliação nacional ao longo do ensino médio. Como um ENEM, mas que fosse aplicado na primeira, segunda e terceira séries. Mas teria que ser um exame de escore único, sem particionamento por áreas e com questões completamente interdisciplinares. Para quem já tenha feito o ensino médio há tempos, seria aplicado um exame global do nível todo. Tem que ser unificado para desconsiderar que algumas escolas são mais frouxas e outras mais apertadas. Por isso as notas que se tira não podem valer. As universidades, por outro lado, não deveriam ter entrada diretamente para os cursos e sim por áreas: humanas, exatas e biológicas. Então se faria um ciclo básico de dois anos sem se escolher o curso. Na virada para o ciclo profissionalizante, se faria uma seleção interna em função das pretensões e das notas no ciclo básico. Como era na Universidade de Brasília à época de sua fundação.

As linhas de força de um campo magnético são 'feitas' de que?‎

Linhas de força são apenas representações e não o campo em si. O próprio campo magnético é uma forma relativística de percepção do campo elétrico. Por isso os dois campos são, de fato, um só, o campo eletromagnético. Tal campo não é feito de nada mais primitivo do que ele mesmo. Ele é um dos constituintes fundamentais do Universo, além dos outros campos e das partículas elementares da matéria. Sua quantização são os fótons. Fótons são fótons. Não são feitos de nada senão deles mesmos.

"A Caixa Preta de Darwin" que está nessa lista não é um livro que tenta derrubar a evolução?‎

Sim, exatamente. Veja sob que tópico eu a listei.

Ernesto, no tempo do Império Romano, mais precisamente no tempo de Constantino, quais eram os povos e civilizações que viviam do lado de cá, nas Américas?‎

Veja isto:
http://pt.wikipedia.org/wiki/Era_pr%C3%A9-colombiana

A filosofia – mesmo sem conotação religiosa – insiste frequentemente no fato de ser ela mesma uma preparação para a morte. Um capítulo dos Ensaios de Montaigne se intitula "Filosofar é aprender a morrer". Aprender a morrer pode tornar a vida de alguém melhor?‎

Filosoficamente, aprender a morrer é o mesmo que aprender a viver. Consiste, justamente, em refletir sobre como se deve levar a vida para que não se tenha temor nenhum da morte. Isso não precisa ter nenhuma conotação religiosa. Mesmo considerando que a morte seja um fim definitivo, como é, pode-se encará-la com serenidade se se levar uma vida significativa, isto é, uma vida de que se tenha consciência de que está sendo proveitosa para si mesmo e para o mundo. Ou seja, que se veja que, ao morrer, se deixa o mundo melhor pelo fato de se ter vivido. Isso é que traz uma paz interior pela qual não se importa de morrer a qualquer momento.

Ernesto, é possível fazer as disciplinas de física geral usando o livro do feynman?(estou refazendo a pergunta, pois acho que você não a viu(como já disse para fazer em alguma resposta do ask)). Mais uma coisa: se pronuncia Ernesto fon Ríquert sendo o R do Ríquert como em cara, ou como em carro?‎

O livro do Feynman é muito avançado para se fazer Física Geral. Além do mais ele não é um livro didático, pois não tem exercícios e problemas. É um ótimo livro para ser estudado como complemento. Mas tem que ter um texto didático. Recomento o de Berkeley ou o Alonso & Finn. Ambos também são bem mais puxados do que os do Halliday-Resnick ou Sears-Zemansky. Além da sequência de conteúdos ser diferente, o que faz com que não fiquem coincidentes com as avaliações. O "R" de "Rückert" é como os "rr" de "carro".

Vc não acha que pelo fato de a burguesia controlar boa parte do estado em relação a escolhas politicas e sociais a implantação do anarquismo aumentaria ainda mais esse imperialismo burguês já q o estado não estaria presente pra intermediar isso ?‎

A anarquia não pode ser implantada. Isso é impossível. A anarquia é uma situação a ser atingida por evolução. Para algo ser implantado tem que haver um comando que controle a ação. E isso é contrário aos princípios anarquistas. E a evolução que levará a anarquia contempla, como característica essencial, a abolição das classes sociais. Mas isso também não pode ser imposto, por uma revolução, por exemplo. Porque em uma revolução há os vencedores e os vencidos. E isso já se configura como classes sociais.

Você é evolucionista? Quais livros recomenda para entender a seleção natural?‎

Certamente que sou. E isso não é uma crença, como alguns dizem. É, simplesmente conhecimento epistêmico. Inescapável. Não ser evolucionista é ser ignorante. Sem apelação. Tão ignorante quanto achar que é o Sol que gira em torno da Terra, como acha o Olavo de Carvalho.
Veja isto:
http://wolfedler.blogspot.com.br/2010/10/qual-o-melhor-livro-sobre-evolucao-que.html
http://wolfedler.blogspot.com.br/search?q=evolu%C3%A7%C3%A3o

Partículas fundamentais são os férmions e bósons. Partículas elementares são os quarks e léptons. Essa nomenclatura está certa Ernesto? Ou Partículas fundamentais e Partículas elementares são sinônimos?‎

Férmions e bósons são uma classificação de partículas que se aplica tanto às elementares quanto às compostas. Férmions são as partículas de spin semi-inteiro que seguem a estatística de Fermi-Dirac enquanto bósons são partículas de spin inteiro que seguem a estatística de Bose-Einstein. Partículas elementares ou fundamentais significam o mesmo, isto é, aquelas que não são compostas por outras mais básicas. Uma grande diferença entre as partículas elementares bosônicas e fermiônicas é que as primeiras podem surgir e desaparecer à vontade enquanto as últimas apresentam conservação do número delas, só podendo ser transformadas em outras, mas não surgir e desaparecer. A não ser que sejam aniquiladas pela correspondente anti-partícula, quando então são transformadas em fótons, que são bósons. Quarks são os férmions, que constituem os hádrons (bárions e mésons), enquanto léptons são os elétrons e seus irmãos pesados, os múons e os táons. Tais são as partículas constituintes da matéria, conjuntamente com glúons e outros bósons que transmitem as interações internas dentro dos hádrons. Não há matéria sem férmions. Bósons puros não são matéria e sim campos e radiação.

Partículas fundamentais são os férmions e bósons. Partículas elementares são os quarks e léptons. Essa nomenclatura está certa Ernesto? Ou Partículas fundamentais e Partículas elementares são sinônimos?‎ Leinadv Férmions e bósons são uma classificação de partículas que se aplica tanto às elementares quanto às compostas. Férmions são as partículas de spin semi-inteiro que seguem a estatística de Fermi-Dirac enquanto bósons são partículas de spin inteiro que seguem a estatística de Bose-Einstein. Partículas elementares ou fundamentais significam o mesmo, isto é, aquelas que não são compostas por outras mais básicas. Uma grande diferença entre as partículas elementares bosônicas e fermiônicas é que as primeiras podem surgir e desaparecer à vontade enquanto as últimas apresentam conservação do número delas, só podendo ser transformadas em outras, mas não surgir e desaparecer. A não ser que sejam aniquiladas pela correspondente anti-partícula, quando então são transformadas em fótons, que são bósons. Quarks são os férmions, que constituem os hádrons (bárions e mésons), enquanto léptons são os elétrons e seus irmãos pesados, os múons e os táons. Tais são as partículas constituintes da matéria, conjuntamente com glúons e outros bósons que transmitem as interações internas dentro dos hádrons. Não há matéria sem férmions. Bósons puros não são matéria e sim campos e radiação. 20 dias atrás9 pessoas curtiram isso Feliz Dia em que Jesus morreu na ceuz do calvario pra nos salvar!‎

Obrigado por me desejar felicidade. Só que Jesus não morreu na cruz para nos salvar. Ele morreu porque, por azar, o povo estava de vento virado quando Pilatos tentou evitar sua condenação. Mas morreu em vão. Quanto a salvar, salvar de quê? De não poder ir para o céu? Ora, mas céu não existe. Então não se pode ir para ele mesmo, com ou sem a morte de Jesus.

Enem ou Vestibular?

ENEN, sem dúvida. Mas o ENEM precisa ser reformulado. Primeiro tem que ter um nível mais elevado de exigência e, depois, tem que ser todo de questões interdisciplinares sem separação por áreas de conteúdo.

Pode-se dizer que um bóson é uma "partícula de campo" e um férmion uma "partícula de massa"?‎

As quantizações de campo sempre são bosônicas, mas os bósons podem ter massa, bem como carga, como acontece com os bósons W do campo da interação fraca. Os férmions sempre têm massa, apesar de que já se considerou que o neutrino não tivesse massa. A matéria sempre requer férmions para existir. Mas pode ter bósons também. A maior parte da massa dos hadrons é devida à energia dos glúons que existem dentro deles.

Situação: Você é casado(a) e o relacionamento está dilacerando, sua terapêutica disse que a melhor opção pra salvar o casamento seria oferecer um "passe livre" pro companheiro(a), e você tem certeza de que daria certo. Ainda assim, faria isso? Deixaria a pessoa que ama livre para outras pessoas?‎

Certamente que sim. Não vejo problema nenhum. Mesmo que o relacionamento esteja bom.

Sobre musíca clássica: prefere as mais calmas ou as mais agressivas?‎

Prefiro as mais dramáticas, mas não tanto agressivas.

Ernesto, você acha que uma mãe que dar seu filhinho por falta de condições para cuidar dele, -condições em todos os sentidos da palavra. Ela está certa em querer o melhor para seu filho, mesmo sendo com outra família? Isso é amor? -O que você acha?‎

Claro que está certa. Claro que isso é amor. É altruísmo.

oi ernesto, sou formado em licenciatura da matemática e estou fazendo pós graduação em matemática financeira, estou meio confuso, gostaria de saber oq seria melhor pra mim, optar pelas finanças ou seguir como docente ?‎

Pessoalmente eu não tenho o menor interesse por finanças. Mas cada pessoa tem seus próprios gostos. Acho que o magistério é uma atividade fantástica, para mim muito mais significativa e capaz de dar sentido à minha vida. Levar conhecimento à juventude é algo maravilhoso. Ainda mais de matemática, que considero uma verdadeira "boatemática".

Por que seria absurdo e ficcional?tem alguma comprovação pra explicar?‎

O maior absurdo de toda a Bíblia é a noção de que Jesus teria que ser morto na cruz para redimir a humanidade. Isto é, Deus ficou irado com o pecado original de Adão e Eva e não permitiu mais que a humanidade pudesse gozar da vida no paraíso. A não ser que se aplacasse sua ira com um sacrifício excepcional, ou seja, a imolação de seu próprio filho em sofrimentos atrozes. Então ele ficou satisfeito e deixou a humanidade poder ir para o céu (se merecesse). Que Deus mais pérfido é esse! Eu não queria ser amigo dele. A ficção fica por conta da história mitológica da criação, de Adão e Eva, da divindade de Jesus, de sua ressurreição e outros episódios completamente ficcionais ( a parada do Sol para Josué vencer a batalha e o dilúvio, por exemplo).

Qual é a sua opinião sobre o feriado da "Sexta-feira Santa"?Que para mim, é apenas um dia qualquer, a única coisa que me interessa, é o feriado, e mais nada.‎

Esse feriado existe porque o cristianismo foi uma força dominante na sociedade ocidental por muito tempo, ainda sendo em boa parte. Então os estados tinham o cristianismo como religião oficial e, nele, a morte de Jesus é o episódio mais importante da noção de "redenção", em torno da qual se estrutura toda a doutrina cristã. Desse modo é a sexta feira da paixão a data magna do cristianismo e não o natal ou a páscoa. Foi com a morte de Jesus que a humanidade ficou redimida, como o cristianismo entende. Quando o cristianismo for abolido como essa força que ainda é, tal feriado será, naturalmente, extinto.

Como pode ter certeza que Deus não existe?Eu tenho certeza de que se você lesse pelo menos um capítulo da Bíblia,iria mudar de ideia.‎

Já li muitos capítulos da Bíblia mais de uma vez. Inclusive a leitura dela foi um dos mais relevantes motivos para que eu deixasse de crer em Deus. Trata-se de um livro completamente absurdo. E inteiramente ficcional.

http://physics.stackexchange.com/questions/106718/the-scissor-paradox-can-we-pass-the-information-faster-than-light Os cálculos desta página procedem?‎

Os cálculos estão corretos e, de fato, o ponto de separação entre uma região que já foi cortada por uma tesoura que corte uma folha de papel e aqueles que ainda não o foram, pode se deslocar ao longo do papel com velocidade maior do que a da luz. Do mesmo modo que uma sombra pode se mover sobre um anteparo com velocidade maior do que a da luz. E isso não contraria, em nada, a teoria de relatividade, pois nem a sombra nem o limite do corte são entidades físicas, mas apenas geométricas. A questão é se se pode considerar que esse movimento seja uma informação sendo levada. A informação em questão seria o fato do ponto ter já sido ou não cortado ou se a luz está ou não batendo no lugar. Pode-se concluir que, quando o corte atingiu um lugar, os lugares anteriores já foram cortados. Mas isso não é uma informação comunicada e sim deduzida por raciocínio. Do mesmo modo que se pode dizer que, uma sombra estando se movendo, se ela chega em dado lugar (e lá pode ter um sensor de luz) é porque já passou pelos lugares anteriores de sua trajetória. Da mesma forma esse é um conhecimento deduzido e não passado pelos lugares por onde a sombra já passou aos lugares onde ela ainda vai passar.

O respeito se impõe ou se conquista?

Se conquista. Se for imposto não é respeito: é medo.

Quando você refuta todos os argumentos de uma pessoa em um debate, e a mesma começa a usar apenas ad hominem em seguida, o que você faz?‎

Explicito que ela está agindo assim e, se ela prosseguir, recuso-me a continuar.

O que exatamente quer se dizer quando se fala "não é possível enviar informação acima da velocidade da luz"? E por que isso ocorre?‎

Toda informação requer um portador físico para ser levada. E nenhum sistema físico, seja material, seja radiante, consegue se mover com velocidade maior do que a da luz.

Qual a área da física que você mais gosta e a que você menos gosta?

Gosto mais de relatividade e física de partículas elementares. Gosto menos de mecânica clássica e termodinâmica. Mas gosto também.

O que é este site pra sua vida?

Uma oportunidade para divulgar minhas ideias e para esclarecer muita gente a respeito de conhecimentos científicos, filosóficos, políticos, religiosos e outros. Um complemento a minha atividade docente.

Dá pra seu budista e ateu?

O budismo é uma religião ateísta.

Oi, boa noite. As pessoas costumam te tratar diferente por ser ateu?‎

Não. Nunca ocorreu isso. E eu proclamo para todos que sou ateu. Sem o menor problema. Sempre fui e continuo sendo uma pessoa respeitada e considerada por ser quem sou e como sou. Por meu caráter, por minhas ações, por minhas convicções, por meu trabalho, por minhas ideias, por meus sentimentos. Em suma, pela coerência de minha vida.

e aí,eu não acredito em nenhum Deus,mas eu não me rotulo como um ateu,isso faz algum sentido no seu ponto de vista?‎

Não. Mesmo que você não se rotule, se você não acredita em nenhum deus, você é ateu. Portanto, é melhor dizer que é mesmo. Assim não se fica em cima do muro e todo mundo sabe o que você pensa a respeito. Diga e proclame ser ateu, como um cristão deve proclamar que o seja, bem como um judeu, um muçulmano, um budista etc. Ou qualquer outra coisa, como um petista, um democrata, um nazista, um homossexual e assim por diante. Se se é algo, é para se dizer que se é.

Professor, o anarquismo é algo feito de uma hora para outra ?

Anarquismo não é uma situação: é uma concepção e um movimento. Anarquia é que é a situação que o anarquismo pretende que seja estabelecida. Essa situação não se alcança subitamente e sim por meio de uma evolução gradativa, até que se estabeleça sem que se perceba. Isso demanda várias gerações ou alguns séculos, ou mesmo milênios.

Qual postura mais lhe agrada em um debate? Uma postura com respostas mais ácidas e as vezes até arrogantes, tipo Richard Dawkins, ou respostas mais suaves e didáticas, tipo Neil deGrasse Tyson?‎

Não gosto de ser agressivo e mal educado. Sempre trato quem discorda de mim com toda a cortesia. Mesmo que essa pessoa não seja cortês para comigo. Não perco a calma e nem me exaspero. Sou fleumático. Mas sou persistente e obstinado. Com a força da mansidão. Busco o convencimento por argumentos.

"Depois que existe tempo e espaço e o Universo, em verdade, não existe mais o surgimento de coisa alguma". Os exemplos de eventos sem causa, como o decaimento radioativo, não é um "surgimento"?‎

Não. É uma transformação. Os produtos da radioatividade não são originários de nada. O que esse evento não tem é causa. Mas ele se dá em atendimento às leis de conservação.

O que acha disso? https://www.youtube.com/watch?v=nX1b6jOFvlo‎

Isso é uma bobeira sem tamanho. Ser excomungado não tem o menor significado. Não tem importância nenhuma. Somente a pessoa deixa de fazer parte efetiva da Igreja. Ora, qual o problema de não fazer parte da Igreja? Por outro lado, ser comunista é uma posição econômica. Não tem nada a ver com pertencer ou não a alguma religião. Ou em acreditar ou não em deuses. O que acontece é que os ideólogos do comunismo (que nunca o implantaram em lugar nenhum) consideravam que as religiões eram empecilhos para a libertação da sociedade do jugo dos poderosos. No que tinham toda a razão. Portanto, a maior parte de quem se filia às ideias comunistas também considera que religião é um entrave ao progresso. Ora, e que tem isso de mais? Isso é que é muito bom. As religiões precisam acabar mesmo. Independentemente do mundo se tornar comunista ou não.

Deus te ama.‎

Claro que não, pois o que não existe não pode amar.

Você acha que um ateu deve comparecer à missa de sétimo dia de um parente?‎

Claro que sim. Em atenção, consideração e respeito por seus familiares que professem a religião do culto em questão. Ser ateu não significa ser fanaticamente anti-religioso. Nem significa depreciar quem tenha alguma religião.

http://ask.fm/wolfedler/answer/125672693533 Não seria um elipsoide?Ou é a mesma coisa?‎

Elipsoide é um caso particular de esferoide. Qualquer esfera achatada ou esticada é um esferoide. Pode ser que esse achatamento ou esticamento seja de modo que a seção máxima tenha a forma de uma elipse. Então será um elipsoide, oblato (achatado) ou prolato (esticado).

Para algo surgir, mesmo por acaso, não é preciso tempo? Como o universo pode ter surgido se não havia tempo?

Não é preciso tempo prévio para algo surgir. O evento único e singular do surgimento do Universo é o evento do surgimento do próprio tempo, junto com o espaço e o conteúdo do Universo. Depois que existe tempo e espaço e o Universo, em verdade, não existe mais o surgimento de coisa alguma mas apenas a transformação de algo em algo diferente. Isso acontece ao longo do tempo e dentro do espaço. Mas o próprio evento do surgimento do espaço e do tempo não ocorreu dentro do tempo e do espaço.

Por que é que você usa clichês do tipo "se Deus existisse, não permitiria tal coisa pois diz ser o amor e justiça"? Ora, se esse plano de vida não é o principal, segundo Ele, e a verdadeira vida venha após a essa, então por que se valer desses sentimentalismos anti-cristãos egoistas?‎

Eu não uso nenhum clichê desse tipo. O que eu não sei é de onde você tira essa informação de que Deus tenha dito que a verdadeira vida virá depois desta. Não me consta que Deus tenha dito nada sobre nada. O que me consta é que pessoas disseram o que achavam que seria o que Deus diria, na opinião delas, e passaram isso como se Deus é que o tivera dito. E que sentimentalismo egoísta anti-cristão você se refere? Que eu seja anti-cristão é fato, pois o cristianismo é um engodo. Não só ele, como todas as religiões e crenças religiosas. Considerar, como considero que, se Deus existir, ele seja malvado é anti-cristão sim, mas não é sentimentalismo nenhum. É uma constatação a partir dos fatos, como já o bem dissera Epicuro.

Se a Terra é esférica, por que o paralelo da linha do Equador é mais extenso que os meridianos?‎

Porque ela não é esférica e sim um esferoide oblato. Seu raio equatorial é maior do que o raio polar. Assim o equador é o maior círculo máximo que se pode traçar sobre ela. Os meridianos são menores porque à medida que se vai do equador ao pólo o raio deles vai diminuindo, enquanto o do equador fica sempre o mesmo, exatamente o maior deles. Leia isto:
http://pt.wikipedia.org/wiki/Figura_da_Terra

Existe um debate a respeito da conceitualização da sociedade. Ontologicamente, você diria que a sociedade é um ente?‎

Sim, é um ente. Todas as instituições são entes. O que não é ente são os valores, as ações, as relações, os fenômenos, os atributos (energia, por exemplo). Números também são entes. Mas conceitos não são entes. Figuras geométricas também são entes. Funções, funcionais e operadores são entes. Sistemas físicos são entes. Campos de força são entes. Radiação é um ente. Objetos são entes. Animais, vegetais, fungos, bactérias são entes. Pensamento não é um ente. Sentimento não é um ente. Emoção não é um ente. Und so weiter.

http://ask.fm/wolfedler/answer/125678247453 ... Muitas ideias da "física moderna" vão ser consideradas, num futuro próximo, mais fantasiosas que a estória da torre de babel, das pragas do Egito, da abertura do Mar Vermelho, das muralhas de Jericó, da divindade de Cristo etc.‎

Não é assim não. Esses fatos citados são mitológicos, isto é, invenções, ficções. Os conhecimentos de Física, Biologia, Geologia, Astronomia, Cosmologia, Química, Sociologia, Psicologia, História e das demais ciências, por outro lado, são fundamentados. É claro que eles vão sendo aprimorados e substituídos por outros mais corretos, à medida que o conhecimento avança. Isso não significa que o que se sabia antes era errado e sim que, face aos dados disponíveis então, era o que correspondia à verdade. Quanto à Filosofia, ela também revê suas assertivas à medida que a realidade vai sendo cada vez mais conhecida em abrangência e profundidade. Isso é normal. Para que uma pessoa possa dizer que entende o mundo em que se insere, tem que saber matemática, ciência e filosofia sim. E, das ciências, a mais fundamental é a física. Pena que o Ensino Médio não aborde Física Moderna. Há mais de quarenta anos que me bato por isso, em vão. Só na EPCAR é que tive o prazer de lecionar Física Moderna para o nível médio. Nas demais escolas só querem saber do que cai nos vestibulares e, agora, no ENEM. Uma lástima.

Por que você acha que é preciso saber Matemática pra se entender o mundo no qual estamos inseridos? Física me parece óbvio e concordo. Mas Matemática me parece uma lógica abstrata, que não diz respeito ao mundo natural. Por que todos deveriam saber? Me parece mais plausível Física e Filosofia.‎

Porque sem matemática não se entende as relações entre as entidades constitutivas do mundo. Como umas influenciam as outras. Não saber matemática é mais condenável do que não saber gramática, por exemplo. Acho estranho que muitas pessoas consideram inculto alguém que erre o português ao falar mas não alguém que não saiba somar frações. Ou calcular a área de um trapézio. Do mesmo modo que toda pessoa que possua um nível superior de educação tem que saber história e geografia, senão é um ignorante, também tem que saber matemática, física, química e biologia, senão é um ignorante. O que se ensina no nível médio, em todas as matérias, tem que ser sabido por todo mundo que tenha curso superior, seja de que área for. Não só engenheiros têm que saber português e história, como médicos têm que saber matemática e geografia e advogados têm que saber física e biologia. No nível elementar do Ensino Médio, bem entendido.

(continuação) Como você acha que se poderia fazer para driblar esses termos inapropriados, que podem parecer pequenos para alguns mas fazem toda a diferença, na divulgação científica, e ao mesmo tempo explicar com precisão, de forma concreta?‎

Isso é simples. Basta ser rigoroso nos conceitos e no relato das relações entre os fatores que influem nos fenômenos. Nunca dizer nada errado para não complicar. Se for preciso, que se complique. Mas de uma forma didaticamente compreensível. Sempre se pode fazer assim. Por exemplo, no caso da massa e energia, não dizer que massa é energia e sim que, em uma reação nuclear, parte da massa dos reagentes pode ser convertida em energia dos produtos por meio da relação E = mc². Da mesma forma que, quando várias partículas se unem para formar um sistema composto, a massa desse sistema inclui as energias de ligação das partículas que o constituem. É isso. De modo correto e claro. Pior ainda é dizer que matéria é energia ou que energia é uma entidade constitutiva do Universo. Energia e massa são atributos dos sistemas e não entidades. As entidades constitutivas do Universo são matéria, radiação e campo. Que podem possuir massa e energia, como propriedades.

Boa noite Professor! O que acha acha da Astrologia?‎

Uma grandíssima bobagem. Sem o menor fundamento. Completamente inconclusiva e totalmente errada em suas previsões.

http://ask.fm/wolfedler/answer/125675370269 ****Matemática e Filosofia.‎

Concordo em acrescentar a Filosofia. Mas não em retirar a Física. Só que, para mim, o importante não é conhecer Filosofia, mas sim saber filosofar.

Professor, quanto mais estudo, mais me sinto "burro". Me desanima... Isso acontece com você?‎

Não. Quanto mais eu estudo, mais tenho vontade de estudar e saber mais ainda. Acho fascinante o conhecimento. É uma cachaça para mim. Não só o conhecimento mas as habilidades em usá-lo. Sigo o pentagrama: estudar, pensar, saber, sentir, fazer. Alguns acrescentam a sexta ação: vencer. Penso, contudo, que vencer não é importante. Nunca tenho a sensação de ser burro. Mas tenho a sensação de ser cada vez mais ignorante, à medida que mais sei. Porque, então, tomo cada vez mais consciência do quanto que eu não sei.

Ernesto, o senhor havia respondido antes sobre os livros do Halliday e tal... Qual coleção o senhor acha a mais indicada para estudantes que queiram se preparar para um mestrado em física? Berkeley seria mais aconselhado que Moysés?‎

O Nussenzweig é bom, mas não é melhor do que o Alonso & Finn e, principalmente o Berkeley. Este último, acompanhado das "Lectures" do Feynman são o melhor caminho para se preparar bem para a segunda metade do Bacharelado e, depois, para o Mestrado e o Doutorado. Mas não bastam. Há que se aprofundar no estudo das disciplinas específicas da segunda metade do Bacharelado, como Mecânica Clássica, Eletromagnetismo, Ótica, Termodinâmica, Física Quântica, Física Estatística e, especialmente, os Métodos Matemáticos da Física. Dependendo da área em que for se pós-graduar, é preciso fazer Física Nuclear, Relatividade Geral, Física da Matéria Condensada, Astrofísica, Eletrônica ou outras.

O que você acha que todo mundo deveria saber?

Matemática e Física. Senão não entende o mundo em que está inserido.

https://www.youtube.com/watch?v=mqB80u0qWHw Adoro o trabalho do Gleiser, mas achei uma mancada quando ele falou que "energia é matéria pela equação E=mc²". Esse é o problema em se fazer divulgação científica com simplificações demais não acha? (continua)‎ Marcelo Carsten (continuação) Como você acha que se poderia fazer para driblar esses termos inapropriados, que podem parecer pequenos para alguns mas fazem toda a diferença, na divulgação científica, e ao mesmo tempo explicar com precisão, de forma concreta?

Exatamente. Esse é um grande problema da divulgação científica. Cometer erros como esse por se simplificar demais. Jamais faço isso. Prefiro complicar. Acho que as pessoas são capazes de entender o que seja complicado também, desde que esteja bem explicado. Tanto em minhas aulas quanto no que passo neste Ask, por exemplo, não faço simplificações errôneas. É sempre possível se dizer tudo de modo conceitualmente correto, bem como explanar as relações entre os fatores envolvidos em um fenômeno de forma certa. Mesmo que não seja tão simples.

Usarias um carro que se conduz sozinho ou conduzirias tu?

Não gosto dessa ideia. Prefiro eu mesmo no comando do veículo.

fala de não perder tempo com futilidade e ta ai no ask perdendo tempo com uma futilidade‎

Mas é claro que não. O que faço aqui não é futilidade nenhuma. É parte de meu projeto pedagógico de difusão de cultura e conhecimento. Dedico um tempo precioso de minha vida a esse mister porque considero que seja uma forma muito eficaz de disseminar minhas idéias e de esclarecer a juventude sobre muitos pontos que a sociedade passa uma visão equivocada. Isto é uma complementação do meu magistério em salas de aula.

Então o sr é a favor da maioridade em sua totalidade, certo? Não apenas penal. Mas tem certeza? Com 16 anos, eu e vários amigos eramos bem inseguros ainda, e duvido que pedir empréstimos em bancos, dirigir ou casar seria uma boa ideia.‎

Tenho certeza sim. Com dezesseis anos todo mundo pode assumir todas as responsabilidades de adultos. Se isso for cobrado das pessoas, elas assumirão, sem problemas. É um absurdo se passar tanto tempo da vida sem ser plenamente um ser humano.

"Muitas pessoas alegam que o termo ateísmo deve ser usado somente para essa vertente "forte", sendo correto designar o ateísmo fraco por outros meios como "ceticismo" ou "agnosticismo"." Por essa definição o senhor seria agnóstico?‎

Não sou agnóstico nem gnóstico, sou cético. Isso significa que nem considero que a existência de Deus não possa ser provada e nem descartada e nem considero que se tenha alguma prova de que exista ou não exista. O que considero é que não se tem certeza nem de sua existência nem de sua inexistência. Todavia considero que não exista. Isso é ateísmo sim. Ateísmo cético ou fraco. O ateísmo forte ou gnóstico é o de quem considera que seja certo que não exista Deus, sem dúvida. Não procede considerar apenas o ateísmo gnóstico como ateísmo. E nem chamar o ateísmo cético de agnosticismo. Quem for agnóstico, gnóstico ou cético tanto pode achar que deus exista quanto que não exista. Existem, pois, seis possibilidades: ateu gnóstico, ateu cético, ateu agnóstico, teísta gnóstico, teísta cético e teísta agnóstico.

Professor, li seu perfil e me deu curiosidade : Como consegue se dedicar e como conseguiu fazer tanta coisa importante assim na vida? Admirável!‎

Basta não ter um pingo de preguiça e não perder tempo com futilidades, como ver televisão, além de não dormir demais. Aplicando-se com dedicação a tudo o que faça, não se preocupando se está tendo a devida retribuição ou não, se consegue fazer muita coisa. Em geral, não se tem a devida retribuição mesmo. Mas se se deixar de fazer algo porque não se é bem retribuído, não se faz nada. Quem consegue fazer muita coisa é porque é desprendido mesmo.

Se estivesse olhando para nosso planeta nesse momento, um alienígena situado num planeta a 100 anos-luz de distância enxergaria a Terra em 1915, há exatos 100 anos atrás?‎

Exatamente. Isso se conseguisse enxergar.

Ernesto, cabe a ciência a incumbência de mensurar em que momento um ser humano é adulto para todos os efeitos? Há estudos publicados neste sentido?‎

Biologicamente um animal é adulto quando já é capaz de procriar. No caso humano, isso é postergado porque os humanos são gregários e a maturidade social ainda não está estabelecida lá pelos 13 a 14 anos, quando a sexual é atingida. Essa maturidade envolve, principalmente, a noção de responsabilidade. Não conheço estudos a respeito, mas considero que, aos dezesseis anos, qualquer pessoa já possua responsabilidade para arcar com as consequências de seus atos, por ter plena lucidez a respeito do que elas possam ser. Então, para mim, dezesseis anos é a idade com que se possa considerar alguém adulto para todos os fins. Inclusive para prover-se a si mesmo com seu próprio trabalho, dispensando ser sustentado pelos pais. Ou seja, a pessoa viveria às custas dos pais até a idade de terminar o Ensino Fundamental. Depois faria o Ensino Médio e o Superior, se fosse o caso, a suas próprias custas. Realmente a determinação da idade em que se atinge a maturidade social tem que ser objeto de pesquisa científica em psicologia e sociologia. Mas não conheço estudos a respeito.

Boa noite, por que os livros do Halliday são tão criticados pela grande maioria dos estudantes de física?

Acho que eles consideram que seja um livro puxado, difícil, exigente. Mas é assim que tem que ser mesmo. Geralmente eles se acostumaram com a moleza do Ensino Médio na Física e sentem um baque ao passar para o nível superior. Seria preciso que o Ensino Médio fosse mais puxado e exigente para que o baque fosse menor. Por exemplo, se fosse adotado o PSSC. Em verdade o Halliday-Resnick, bem como seu equivalente Sears-Zemansky, são dois livros de Física Geral de nível pouco avançado, em comparação, por exemplo, com o Alonso & Finn ou o de Berkeley. Outro aspecto é que aqueles são livros mais voltados para engenharia e alunos de Física gostariam de um livro mais voltado para Física. Mas a Física da engenharia é a mesma da Física. É bom, inclusive, que estudantes de Física se familiarizem com aplicações de engenharia da Física. Para um curso de Física, o melhor seria o livro de Berkeley. Mas ele é muito mais puxado do que o Halliday-Resnick ou o Sears-Zemansky.

Muito boa noite Ernesto, sou um rapaz de 18 anos, estou em dúvida se curso física, esse ano faço vestibular, eu sou fascinado no universo e em vários ramos de física, mas o Brasil não é um país que da muito valor para esse ramo, gostaria de lhe pedir conselho e conversar (não perguntar) mais..‎

Até que Física, no Brasil, é uma ciência razoavelmente bem suprida de pesquisadores. O trabalho deles se faz nas Universidades e nos Institutos de Pesquisa. Para tal você tem que ser muito bom, de modo que seja vencedor dos concursos para provimento dos cargos, já que esse é o único meio de alcançá-los. O bom é, durante os cursos, tanto de bacharelado quanto de mestrado e doutorado, já ir se entrosando com os pesquisadores e trabalhando nas pesquisas, bem como publicando artigos, para se garantir de pontos nos concursos. Os pesquisadores, inclusive, são os que fazem as cartas de recomendação e eles recomendam bem quem for, mesmo, bom de serviço.

É ético dar vida a uma espécie bizarra de humano com outro ser vivo em prol de uma experiência transumana de novas espécies? Estou falando do direito de opinião e escolha do futuro ser vivo a ser criado.

Acho que seja, se se fizer, previamente, muitos ensaios e experimentos que garantam que o novo ser não será bizarro, de modo nenhum. É perfeitamente possível se implementar aperfeiçoamentos a várias características humanas, como a sensibilidade dos sentidos, a capacidade de memorização, de raciocínio, do sistema imunológico, do sistema que controla a senescência, para retardá-la ou eliminá-la. E assim por diante. Não vejo problema ético nenhum em se aperfeiçoar a espécie.

Ainda sobre o arredondamento de notas.‎

Sempre que corrijo avaliações dos alunos eu aproveito o máximo que pode ser aproveitado. Então, se algum não atingiu o mínimo exigido, mesmo que faltando só um décimo, não concedo esse décimo de graça porque considero que seja uma injustiça. Injustiça não é só não dar a quem mereça, mas também dar a quem não mereça.

o que o sr acha da redução da maioridade penal‎

Concordo. Esta é uma das concepções direitistas que advogo, mesmo sendo esquerdista. Acho que, a partir de 16 anos, uma pessoa possa ser considerada adulta para todos os efeitos: para dirigir, votar, casar, trabalhar, ter conta em banco, pegar empréstimo e... ser presa.

Por que alterar o DNA humano é visto como tabu nos governos?

Porque muitos governos temem o repúdio da sociedade. E a sociedade repudia porque considera que isso seria vedado por Deus, uma vez que interfere nas características do que venha a ser "humano" e pode levar à fabricação de uma nova espécie, transumana, que muitas pessoas religiosas acham que seja permitido apenas a Deus. Apesar disso, as pesquisas avançarão e, algum dia, se poderá, mesmo, fabricar novas espécies artificialmente. Inclusive transumanas.

Considere dois números naturais, cada um deles tem três algarismos diferentes. O maior deles tem três algarismos pares e o menor deles só tem algarismos ímpares. Se a diferença deles é a maior possível, qual a diferença?

729

Ernesto voltando as perguntas teóricas antigas que eu fazia,Você acha que em dois séculos seremos capazes de conseguir a cura para doenças como Aids,câncer e problemas do coração ?(sua opinião baseado no que sabe,mesmo não sendo especialista no assunto)‎

Sim. Do mesmo modo que já se encontrou a cura de várias doenças, a dessas também será achada. Mas pode demorar. Outra que vai se achar a cura é o envelhecimento.

E essa rapaziada lhe xingando, oque pensa a respeito ? Inveja, admiração ou só falta de educação mesmo ?‎

Falta de educação, antes de mais nada. Mas pode ser despeito.

Professor qual sua opinião sobre as ditas feministas radicais que desconsideram estudo, teorias e afins científicos feitos por homens (chamados por elas de machos)?

Porque elas não são feministas e sim femistas, já que o feminismo considera a igualdade dos sexos. Nem todo homem é machista e nem toda mulher é femista. Há homens feministas e mulheres masculinistas. A pessoa que preza a verdade não é machista nem femista, pois essas são concepções equivocadas. O feminismo, que é o mesmo que o masculinismo, é que são concepções fiéis à verdade. Por isso deveriam ser chamados, simplesmente, de humanismo. Infelizmente, no português, a palavra que designa nossa espécie é a mesma que designa a porção masculina dela. Deveria ser diferente.

Atualmente não leciono mais. Mas, quando o fazia, na UFV, havia, como ainda há, liberdade para o professor adotar o método que quisesse. Sempre gostei de usar o método da redescoberta, isto é, de fazer o aluno ser um cientista e ele mesmo descobrir e deduzir o conteúdo a ser aprendido. O professor fica como um indutor do processo. Os alunos é que trabalham e não apenas assimilam passivamente os conteúdos. Isso faz uma bagunça na turma, mas é ótimo. As avaliações são todas com consulta livre (exceto aos colegas) e sempre verificam a capacidade de raciocinar, especialmente em situações inéditas, apresentadas no momento da prova. O importante não é saber, mas saber descobrir. Claro que, depois de descoberto, é preciso saber e, principalmente, saber usar o conhecimento na solução de problemas reais e não fictícios. Ou seja, os problemas propostos envolvem medições de grandezas em sistemas reais e não dados inventados para facilitar as contas. Os alunos têm que "meter a mão na massa", isto é, medir comprimentos, tempos, velocidades, massas, correntes elétricas, temperaturas, intensidades luminosas e assim por diante. Esses dados é que vão ser usados na solução dos problemas. Além do mais eles têm que saber deduzir fórmulas. Não apenas as que foram apresentadas, mas outras, inéditas. Em suma. é preciso "fundir a cuca".

Sim, desde que não nos importemos que o que dissermos seja contado para o mundo todo.

Professor, como tu preparas e reges suas aulas? segue algum padrão da universidade, ou desenvolve sua maneira de ensinar, e gostaria de saber também se a universidade em que concede suas aulas, lhe fornece liberdade para isso?‎

Atualmente não leciono mais. Mas, quando o fazia, na UFV, havia, como ainda há, liberdade para o professor adotar o método que quisesse. Sempre gostei de usar o método da redescoberta, isto é, de fazer o aluno ser um cientista e ele mesmo descobrir e deduzir o conteúdo a ser aprendido. O professor fica como um indutor do processo. Os alunos é que trabalham e não apenas assimilam passivamente os conteúdos. Isso faz uma bagunça na turma, mas é ótimo. As avaliações são todas com consulta livre (exceto aos colegas) e sempre verificam a capacidade de raciocinar, especialmente em situações inéditas, apresentadas no momento da prova. O importante não é saber, mas saber descobrir. Claro que, depois de descoberto, é preciso saber e, principalmente, saber usar o conhecimento na solução de problemas reais e não fictícios. Ou seja, os problemas propostos envolvem medições de grandezas em sistemas reais e não dados inventados para facilitar as contas. Os alunos têm que "meter a mão na massa", isto é, medir comprimentos, tempos, velocidades, massas, correntes elétricas, temperaturas, intensidades luminosas e assim por diante. Esses dados é que vão ser usados na solução dos problemas. Além do mais eles têm que saber deduzir fórmulas. Não apenas as que foram apresentadas, mas outras, inéditas. Em suma. é preciso "fundir a cuca".

Já pensou sobre a significância do calendário gregoriano? Acha que ele é realmente útil para o ser humano, tirando o fato de ter como quase única finalidade a promoção do comércio?

Acho que essa divisão do ano em meses e semanas é bobagem. As únicas divisões naturais do tempo são o dia e o ano. Qualquer outra é artificial. Não que não possa ser usada, mas não precisa. Para mim os dias deveriam ser numerados sequencialmente no ano de 1 a 365 ou 366. Nada de dias da semana e nem de meses. Tudo funcionaria ininterruptamente qualquer dia do ano e qualquer hora do dia. Com rodízio de trabalhadores, de modo, inclusive, que a razão do tempo não trabalhado para o tempo trabalhado pudesse se maior ainda. Atualmente a lei estabelece que ela seja de 44/168 = 1/7 = 14,3%. Poderia ser de, digamos, 1/9 = 11,1%. As horas também poderiam ser decimais, com o dia tendo 10 horas, a hora tendo 100 minutos e o minuto 100 segundos. Isso daria 100.000 segundos por dia e não 86.400 como é hoje. O novo segundo seria 0,864 do atual. Mas isso é menos importante.

Como distinguir artigos bons de "duvidosos" ?‎

Por sua experiência de vida. Por seu conhecimento do assunto. Pelas indicações dos entendidos. Por seu bom senso.

Linguagem corporal é verídica ou mais uma falácia ?

É verdadeira sim. Veja o livro "O Corpo Fala" de Pierre Weil
http://www.saraiva.com.br/o-corpo-fala-a-linguagem-silenciosa-da-comunicacao-nao-verbal-314078.html

Para a pergunta sobre o Projeto do Nassim. https://www.youtube.com/watch?v=ZbneMcq9eAI‎

Não confio em descobertas alardeadas por propagandas. Isso me cheira a charlatanismo. Ainda mais que no próximo vídeo ele insere Deus nas considerações. Além de levar em conta fatos esotéricos. Realmente não parece coisa séria. Parece até o Amit Goswami. Mesmo assim vou olhar, com bastante pé atrás.

Fiquei surpreso com o post ''revoltado'' sobre os físicos (e a Física) de um suposto matemático(?), principalmente porque estas aéreas do conhecimento são tão frutíferas quando andam juntas. Acho que alguns matemáticos sofrem de um complexo de inferioridade, concordando com as palavras do grande‎

matemático V.I. Arnold (que, por sinal, sabe muita Física). E isto implica, muitas vezes, em não terem nenhuma familiaridade com a Física. Esse texto, em minha opinião, é excelente :http://pauli.uni-muenster.de/~munsteg/arnold.html
Obrigado pelo texto. Vou ler. Estudei Mecânica Clássica pelo livro do Arnold, juntamente com o do Goldstein.

Professor, você arredondava as notas de seus alunos para ajuda-los? Digamos que alguém tirou 5,9 em uma prova, você daria um décimo para ele ficar na média?‎

Não. Acho que essa média de 60% é ridiculamente baixa. Não consigo conceber que uma pessoa mereça ser aprovada sabendo apenas 60% do que precisa saber. Para mim teria que ser 90%. Já pensou que um médico pode receber um diploma e exercer a medicina sem saber 40% do que precisa saber para ser médico? E se ele estiver fazendo uma cirurgia e depois que abriu a barriga do paciente vê que o problema se enquadra nos 40% do que ele não sabe?

Na sua opinião, o que você acha que atrai mais as mulheres ?‎

A personalidade, o charme, a elegância, a cortesia, a gentileza, a boa educação, a inteligência, a cultura. Claro que também a beleza, a sensualidade, o vigor. Mas penso que esses últimos atributos em uma pessoa bronca e mal educada não a farão atrativa. Talvez para uma transa, mas não para uma companhia permanente e um relacionamento estabelecido.

Jesus foi um anarquista?

Não considero que tenha sido. Ele não propunha a abolição do governo. O que ele propunha é que as pessoas levassem a vida voltada para a salvação de suas almas e a glória de Deus, desligadas do aspecto político. Que fizessem o bem, ajudassem os pobres. Mas não que se rebelassem contra o governo, que não pagassem impostos ou coisas assim. Os primeiros cristão viviam comunitariamente. Mas não era uma situação a ser estendida à sociedade como um todo.

voce teria como citar artigos da wikipedia(americana) confiaveis que o senhor ja leu sobre fisica‎

Não dá. Foram centenas. Já não sei mais quais foram. A cada situação eu busco e leio. Você deve fazer assim também. E veja que um puxa outro, nas citações. Bem como outros sites a respeito.

Junk food existiria numa sociedade comunista?‎

O fato de ser comunista não cria nenhuma objeção ao consumo de qualquer tipo de alimento, seja saudável ou não. Mesmo num comunismo anarquista, esse tipo pode existir. O que evitaria sua existência seria uma conscientização da população em relação ao cuidado com a saúde. Isso deveria ser buscado, mas não faz parte do conceito do que seja comunismo e nem do que seja anarquismo. Do mesmo modo que as preocupações ecológicas. Note que não estou dizendo que preocupações ecológicas e nem com a saúde possam ser desprezadas ou relegadas a segundo plano. De modo nenhum. Elas são de extrema importância e precisam ser consideradas com toda a seriedade. Só estou dizendo que comunismo e anarquismo não dizem respeito a tais questões. Como também não se relacionam com a questão de haver ou não religião ou crenças religiosas (o que não é o mesmo). Mas eu acho que seria preciso que, a par do comunismo e do anarquismo, também se buscasse, na sociedade, o ateísmo, o ecologismo e a saúdismo.

Tenho uma amiga, e uma namorada... Amo ambas igualmente, incondicionalmente... (Não desejo ter relacionamento a três... Apenas amo as duas, assim como amo as pessoas de minha família)... Mas minha namorada acredita que eu a deveria amar mais do que alguém que é "apenas amiga"... Qual sua opinião?‎

Não se pode exigir a intensidade do amor de ninguém. Quando se ama alguém é claro que se deseja ser amado por esse alguém em retribuição. Mas amor não pode ser cobrado, pois não é objeto de barganha por nada. Amor não se vende, não se compra e não se troca. Amor só se dá e se recebe. De graça. Sem nenhuma cobrança. Sem nenhuma necessidade de retribuição. Portanto sua namorada não pode cobrar amor de você. Muito menos querer que você não ame a quem você ama. Ela tem que entender como é o amor. E não se deve, também, amar ou deixar da amar a quem quer que seja porque outra pessoa queira isso. Ama-se a quem se ama porque se ama e isso é algo sobre o que não se tem controle voluntário. Pode-se perfeitamente amar a mais de uma pessoa com qualquer tipo de amor. E pode-se, também, amar com amor platônico, isto é, que não envolva relacionamento amoroso direto. Pode-se ter um relacionamento puramente amistoso com uma pessoa a quem se ame por qualquer que seja a razão. E, paralelamente, amar-se, também, a outra pessoa com a qual se tenha relacionamento amoroso (namoro ou união conjugal mesmo). A respeito do amor, todas as possibilidades são aceitáveis.. E a respeito desse tipo de assunto, várias formas de relacionamento são possíveis: amor, sexo, namoro, relação informal, relação estabelecida, relação legal e assim por diante. Pode-se ter cada uma delas sem as outras ou qualquer combinação delas.

Você gosta de conversar com pessoas desconhecidas? Geralmente você puxa conversa?

Não gosto especialmente e nem desgosto. Não puxo conversa só para conversar, mas acolho qualquer iniciativa de conversa de quem quer que seja. Se algum fato der azo a que uma conversa seja estabelecida, não me furto a iniciá-la.

Professor, curso Engenharia de Materiais e no futuro pretendo trabalhar com pesquisas físicas, principalmente na área de altas energias, construindo detectores, peças de aceleradores e tudo o mais. Para isso devo fazer mestrado e doutorado em Física ou em outra área da engenharia específica pra isso‎

Para trabalhar com o projeto de dispositivos de equipamentos de alta energia, realmente é preciso entender de física. Mas é melhor fazer o doutorado na área específica da engenharia. Dentro do mestrado e do doutorado nessas áreas é dado o conhecimento de física que é necessário.

Física nuclear é só teoria ou tem calculo também?‎

Tem tanto teoria quanto experimentos quanto cálculos. E muito mesmo.

Você prega que o ciúmes é algo desnecessário e deve ser combatido, mas se eu não tiver ciúmes as mulheres vão julgar que eu não ligo para elas, que postura tomar diante disso?‎

Se alguma mulher achar que o fato de você não ter ciúme (e não ciúmes) dela seria uma demonstração de falta de amor, largue-a de lado e encontre outra que entenda como o ciúme é horrível. Ou diga a ela que é assim mesmo que você é e que, se ela quiser manter algum tipo de relacionamento com você, tem que te aceitar como você é.

Os dinossauros, se não fossem extintos, poderiam evoluir para algo parecido com a gente?‎

Provavelmente não. As aves é que evoluíram a partir de pequenos dinossauros. Os mamíferos vieram de répteis, mas não de dinossauros. Possivelmente de algum tipo de lagarto.

“Devia ter arriscado mais, e até errado mais ter feito o que eu queria fazer...” Comente sobre isto.‎

Quando se percebe que vai se viver menos do que já se viveu, começa-se a sopesar o que se fez na vida e se conclui que se deixou de fazer muito que seria valioso por vários receios e insegurança. Por medo de correr riscos. A maturidade mostra que é preferível correr riscos do que estar seguro, mesmo que isso possa levar a retumbantes fracassos. A questão é que o sucesso não é importante. O que importa é fruir a ventura de viver. A vida existe para ser vivida e não para se garantir a prosperidade. Tal conclusão é fruto da sabedoria da idade. Os jovens, por incrível que pareça, são mais conservadores. Depois de velho é que se conclui que se deveria ter sido mais ousado, pois o que importa é o caminho e não a chegada. O fim da vida é o mesmo para todos: a morte. Então que se vá a ela vivendo-se em plenitude, especialmente amando mais, sem ficar preocupado em achar o amor certo.

Caro Ernesto, sobre sua resposta anterior, Bohr estaria certo quanto à física quântica, é isso? – ou isso era do Heisenberg?

Sim. A questão é a interpretação probabilística e indeterminista da física quântica, defendida por Bohr e contestada por Einstein. Heisemberg também concordava com Bohr, mas este foi o mais aguerrido defensor da interpretação probabilística. Apesar desse descordo, eles eram amigos.

Por que você acha que ser um expert em física facilita uma postura polímata? Você disse algo como: "Pra mim é mais fácil, enquanto físico, estudar qualquer outra coisa, mas o inverso é bem mais complicado." Por que?‎

Entender Física requer uma abertura de horizonte cognitivo bem ampla. Pois a Física é a base de tudo o que acontece no mundo. Todo fenômeno, seja químico, biológico, psicológico, sociológico, no fundo, acontece em razão de fenômenos físicos. Então o físico tem que perceber tudo que há de físico neles. Além disso, o tipo de raciocínio que leva a Física a elaborar suas leis é o mesmo que se aplica a qualquer outro ramo de conhecimento. A Física é o protótipo de qualquer ciência. É o exemplo acabado do que seja uma ciência. Outro aspecto importantíssimo é que o modelamento físico da realidade faz uso extremo da matemática, fazendo com que todo físico tenha que ser, também, um matemático. O treinamento de raciocínio que o físico tem o habilita a qualquer outro raciocínio. Pois a investigação científica em física é como um trabalho de detetive policial. É algo fascinante. A construção da física é uma aventura extremamente instigante que possibilita um grande aprimoramento da inteligência e um grande desenvolvimento da criatividade, muito paralela à criatividade artística. Outras atividades não requerem o concurso de tantas habilidades quanto a de físico. Por exemplo, um economista tem que ter alto entendimento matemático, mas só em álgebra e análise e não em geometria. Um médico quase não precisa entender da álgebra e análise. O físico, por sua vez, se imiscui em todos os assuntos matemáticos e outros, não matemáticos também. Como a filosofia. Física é a ciência mais filosófica de todas. Porque cuida de entender o funcionamento mais profundo da realidade.

Mecanica quantica é muito complicado de aprender? Quais são os requisitos?‎

Depende de cada pessoa. Física Quântica é uma abordagem menos matematizada e mais fenomenológica, sendo mais acessível a quem possua apenas conhecimentos de Física Geral introdutória dos cursos superiores da área de ciências exatas. Já a Mecânica Quântica é uma abordagem bem matematizada e requer um treinamento, que é dado, nos cursos de Física, pela disciplina Métodos Matemáticos da Física, que tem os quatro cálculos como pré-requisito. A Mecânica Quântica pede, também, a Física Quântica como pré-requisito, que costuma ser chamada de Estrutura da Matéria. Em nível médio é possível, também, se fazer uma abordagem bem introdutória mesmo, que requer só os conhecimentos de Física Clássica e a Matemática do curso Médio. Na EPCAR, em Barbacena, onde lecionei, o último semestre da terceira série do segundo grau da matéria Física era todo dedicado à Física Moderna, em que se via a Física Quântica, a Física Atômica, a Física Molecular, a Física Nuclear, a Física do Estado Sólido, a Física de Partículas Elementares, a Física Estatística e a Teoria da Relatividade Restrita. Nada disso cai no vestibular, mas como a escola era militar e não seguia nenhum programa do Ministério da Educação, era dado. Só lá é que pude ver isso. No Brasil, nos cursos de graduação da área de ciências exatas, como as engenharias todas, não se dá esse assunto na Física Geral, como acontece no exterior. Acho, realmente, uma lástima. Nem os engenheiros ficam sabendo como o mundo funciona. Só quem for fazer curso de Física e, em alguns casos, de Química. Os biólogos não vêem nada disso. Nem médicos. Nem advogados, Nem administradores, Nem economistas. Acho uma falha extremamente grave do sistema educacional deixar a maior parte da população sem ter conhecimento dos fundamentos básicos do funcionamento da natureza. Outro assunto que poderia ser abordado no nível médio é a Cosmologia e a Astrofísica, bem como Geologia e Meteorologia. Tudo dá para ser visto se se quiser e se deixar de preocupar se cai ou não no ENEM e nos vestibulares.

é necessário saber calculo I,calculo II,calculo III,calculo IV para poder estudar economia?‎

Não. Só os cálculos I e II.

"O Poder do Subconsciente", de Joseph Murphy, foi o livro que mais me pediu para ser queimado. Em verdade, quase incendiava-se por conta própria em minhas mãos, dado as suas "propriedades inflamáveis". A você, algum livro já te implorou para ser queimado, dado a irrelevância do mesmo para o mundo?‎

Não acho que nenhum livro deva ser queimado, por mais idiota que seja. É um perigo muito grande queimar livros. Muito maior do que deixar que besteiras sejam disseminadas. Coisas irrelevantes são combatidas com a disseminação de conhecimentos relevantes, a tal ponto que o povo todo seja tão culto que não aceite irrelevâncias. Então elas desaparecerão por falta de demanda. Inclusive quem pretenda difundir esse tipo de conhecimento, num mundo totalmente culto, se verá tão desambientado que nem tentará. Temas como astrologia e similares serão totalmente extintos quando o conhecimento científico e filosófico for inteiramente disseminado. Para isso é que dedico a minha vida.

já ouviu falar da Gnosis?‎

Sim. Trata-se de uma imensa bobeira. Qualquer conhecimento esotérico é falso. Não há nada disso de conhecimento de fatos transcendentais por meios misteriosos. Não existem fatos transcendentais. Tudo é natural e passível de ser conhecido pelos meios ordinários de aquisição de conhecimento: estudo, pesquisa e reflexão.

http://ask.fm/joelpinheiro85/answer/124920825075 O estupro é um ato libertário?‎

De modo nenhum. Praticar o estupro como forma de libertação do condicionamento antissexual das religiões é um completo equívoco e uma rematada estupidez. O que é preciso é se libertar das amarras antissexuais das religiões pela prática do sexo livre, mas totalmente consentido pelos praticantes. Praticar sexo sem o consentimento de um dos envolvidos é um ato de violência sem o menor cabimento.

Quero saber o que você acha desta citação de Erwin Schrödinger, o grande físico que ganhou o Nobel em 1933: "A imagem que a ciência dá sobre o mundo ao meu redor é muito deficiente. Ela fornece muitas informações factuais, põe toda a nossa experiência em uma magnífica ordem coerente, mas (continua)‎ Nem ir nem vir nem permanecer (continuação) permanece assustadoramente silenciosa em relação a tudo o que está verdadeiramente perto de nosso coração, tudo o que verdadeiramente conta para nós".

Concordo com ele, mas não acho que isso seja uma falha da ciência. Apenas esse não é o propósito dela. Dentro do que ela se propõe, que é uma explicação do funcionamento do mundo exterior, ela vem se saindo muito bem, de modo que se pode confiar que o que ainda não foi explicado pela ciência um dia o será. Quanto às questões existenciais, ainda não se tem um ramo da ciência que cuide cabalmente delas. Quem cuida delas é a filosofia. Ela sim, se debruça sobre essas questões e busca respondê-las. Isso também é cuidado pelas religiões, mas estas partem de pressupostos equivocados, de modo que não são de confiança. A psicologia seria a ciência que cuidaria disso, mas ela ainda não se livrou de sua divisão em escolas conflitantes, de modo que, também, não se pode ter uma explicação e uma orientação psicológicas seguras sobre as questões existenciais, isto é, algo que seja estabelecido com segurança de forma independente da opção por qualquer escola de pensamento psicológico.

É correto afirmar: "a mecânica quântica força-nos à abandonar o realismo ingênuo"? Por quê?‎

A questão é que a física quântica expõe as relações entre o que seria a estrutura da "coisa em si", ou seja, as partículas subatômicas, seus campos de interações e as radiações que emitem e as percepções que a mente faz do que essa realidade passa para ela através dos sentidos. A física quântica também provê modelos explicativos do funcionamento neuronal que constrói as percepções. Com isso fica patente que, mesmo havendo uma relação entre a percepção e a realidade exterior, essa relação não é de coincidência. Um exemplo típico é a cor apresentada por algo, que não é uma propriedade dele mesmo, mas de sua interação com a luz que o ilumina, bem como da percepção mental da luz envidada ao olho, em contraste com o que se percebe de seu entorno.

Que argumentos vc daria para esclarecer quem acha que filosofia é ciencia e portanto o certo e o errado seria uma questão cientifica?‎

Filosofia não é ciência porque sua forma de validar suas assertivas é outra. Enquanto a ciência se baseia na verificação fática, a filosofia se vale da reflexão e da discussão de idéias. A ética, que cuida do que seja certo ou errado nas ações de seres conscientes, não é científica porque os princípios em que se baseia para rotular eticamente alguma ação não se fundamentam em experimentos e sim em reflexões. Mas ela pode se valer de observações do comportamento de quem sofra as ações para aferir se a ação tenha sido benéfica ou maléfica. Inclusive fazendo uso de levantamentos estatísticos. Sempre que for possível se socorrer de dados científicos a filosofia pode e deve acatá-los, sem descaracterizar suas assertivas como filosóficas.

Você já visitou o IAG? Comecei a estudar meteorologia lá esse ano e acho muito agradável e prazerosa a experiência de conviver com pessoas que estão dispostas a descobrir mais sobre o universo. Acho que você ficaria encantado pela biblioteca que temos lá.‎

Ainda não fui lá. Deve ser bem estimulante mesmo. Como no CBPF, onde estudei. Já visitei, em São Paulo, o IFT e o Depto de Física da USP, onde fiz dois cursos de férias.

Os livros de física do Brasil (e traduções), que abordam o eletromagnetismo são quase sempre o volume 3 e os de termodinâmica e oscilações são o volume 2, mas no exterior é ao contrário. Por que é assim?‎

Não me consta que no exterior seja o contrário do Brasil. Os livros adotados aqui, que são traduções de livros estrangeiros, seguem a sequência: Física 1 - Mecânica, Física 2 - Fluidos, Calor e Ondas, Física 3 - Eletricidade e Magnetismo, Física 4 - Ótica e Física Moderna. Em qual deles a ordem da 2 e da 3 é invertida? O único que conheço desse modo é o Curso de Física de Berkeley.

Prof, qual a tua opinião sobre formar num curso tecnólogo visando conseguir o diploma mais rápido para poder fazer concursos de nível superior?‎

Se você está com pressa, pode fazer isso. Mas os empregos de nível superior técnico são menos bem remunerados. Acho melhor fazer um curso pleno para tentar colocações mais bem remuneradas.

Professor, o que o senhor acha da ordem DeMolay?‎

Não gosto desse tipo de associação, como também da maçonaria, dos rosacruzes e similares. Elas se assemelham a uma espécie de religião. Existem conhecimentos restritos apenas aos iniciados. Isso não é bom. Todo conhecimento tem que ser disponibilizado para todos. Acho que se pode constituir grupos de pessoas reunidas em torno de algum projeto, Mas não gosto de associações regulamentadas e instituídas. Gosto de tudo espontâneo e sem regras. Sem afiliação, sem diretoria, sem estatuto, sem atas, sem contabilidade, sem uniformes, sem paramentos, sem templos, sem rituais. Só as pessoas e seu desejo de pensar, sentir e agir.

Na física quântica, qual é a definição mais correta dos "transportadores das forças"? E o que são eles?‎

Transportadores de forças são as partículas provenientes da quantização dos campos de força correspondentes. No caso dos campos elétrico e magnético eles são os fótons. Tratam-se de partículas bosônicas, isto é, podem surgir e se aniquilar, não exibindo conservação de sua quantidade, como as fermiônicas, que constituem a matéria. Além dos fótons existem os glúons que transmitem a força nuclear forte entre quarks e entre hadrons (bárions e mésons) e as partículas W e Z, que transmitem a força nuclear fraca. Também se cogita da existência dos grávitons que transmitiriam a interação gravitacional. Todavia ainda é questão em aberto se a gravitação seja mesmo uma interação.

A MAIORIA dos estudantes de física são arrogantes, prepotentes e se acham melhor do que todo mundo. Não fosse a matemática, não existia a Física. A Matemática é a rainha das Ciências! A coisa mais fantástica que a humanidade já criou, depois da música. Matemática e Filosofia>>>>>resto.‎

Sou formado em matemática e adoro matemática. Mas gosto muito mais ainda de física. Não acho que a maioria dos estudantes de física sejam arrogantes. Só alguns. É importante saber que grande parte da matemática foi desenvolvida por físicos para resolver problemas de física. Quase todo físico, pelo menos os teóricos, sabe muita matemática. Às vezes mais do que matemáticos. E raros matemáticos entendem muito de física. Em geral, os físicos são os cientistas que mais entendem de outras ciências também. A razão de eu ter me dedicado profissionalmente à física e não à matemática foi justamente que, como físico, eu poderia entender tanto de matemática quanto um matemático. Além de conseguir, também, entender de vários outros assuntos, como filosofia e música, por exemplo. E, como músico ou filósofo, dificilmente eu conseguiria entender tanto de física. Como físico eu consigo entender muito mais de biologia do que um biólogo de física, de modo geral.

É verdade que os gravitons não tem a existência comprovada?‎

Sim. Jamais foram detectados.
http://en.wikipedia.org/wiki/Graviton

Pessoas sentimentais podem se centrar em si mesma , um narcista podem ser extremamente sentimental e não se importar com os outros. supervalorização de si mesmo tbm é sentimentalismo só que de maneira negativa,‎

Pode. Mas não necessariamente. Narcisista é sentimental mas sentimental não precisa ser narcisista. Ser sentimental não implica em supervalorizar-se, mesmo que quem se supervaloriza, geralmente seja sentimental. Quanto a se importar com os outros, o que acontece é que quem seja sentimental, justamente, é uma pessoa que, quase sempre, se preocupa com os outros. Mesmo que alguns não o sejam. Em outras palavras: o fato de haver pessoas sentimentais que o sejam de modo negativo não desqualifica o sentimentalismo.

Escrever, mesmo que de maneira "ruim", nos ajuda a melhorar a nossa escrita, da mesma forma que ler ajuda a melhorar nosso vocabulário? Ou existe maneiras mais eficazes de se melhorar a escrita?‎

Não. É assim mesmo. Só se aprende a escrever escrevendo e lendo muito. Como se aprimora nadar, nadando. Todavia sempre é bom estudar gramática. Mas sem praticar, não adianta. Estou dizendo escrever, pelo menos, uma hora por dia. Mais, se puder.

Em uma resposta, o senhor me disse ser anarco-comunista (comunista em relação com a posição economica). Qual sua opinião sobre esse vídeo? Fala sobre a economia comunista. http://youtu.be/9a8axWVb-2E‎

Acontece que ele está confundindo comunismo com economia estatizada. O socialismo de estado não é comunismo. Por outro lado, o argumento de Mises se baseia em uma economia monetária. A economia não precisa ser monetária. Nem precisa ser de escambo. Pode ser de doação. E mercado não precisa ser de compra e venda e nem de troca. Pode ser de distribuição e captação. Isso é que é comunismo, e não o que aconteceu na União Soviética. Ainda não houve nação comunista em lugar nenhum. No comunismo os meios de produção e a produção não são do estado. São da população. No socialismo estatal é que são do estado. No comunismo os trabalhadores não são empregados nem funcionários. Isso acontece no socialismo de estado. Carvalho se refere é à economia socialista e não à comunista.

Utilizaria o direito a autodefesa para matar alguém? Em quais circunstâncias?‎

Sim, se isso for inevitável para proteger minha vida ou minha integridade, bem como de pessoas inocentes.

LinkWithin

Related Posts with Thumbnails